66

COLLECTION DURANDEAU SCIENCES

  • Upload
    others

  • View
    3

  • Download
    0

Embed Size (px)

Citation preview

Page 1: COLLECTION DURANDEAU SCIENCES
Page 2: COLLECTION DURANDEAU SCIENCES

COLLECTION DURANDEAU

SCIENCESPHYSIQUES ET CHIMIQUES

Livre du professeur

2dePROFESSIONNELLE

BAC PRO

J-P. DurandeauJ-L. BerducouJ-C. Larrieu-LacosteC. MazeyrieC. RaynalJ-C. Trillaud

SP-LP-PT 12/05/09 16:55 Page 1

Page 3: COLLECTION DURANDEAU SCIENCES

Table des matières

1 Quelle différence entre température et chaleur ? CME1 . . . . . . . . . . . . . . . . . . . . . . . . . . . . . . . . . . 4

2 Quels courants électriques dans la maison ou l’entreprise ? CME2 . . . . . . . . . . . . . . . . . . . . . . . . 9

3 Comment protéger une installation électrique ? CME2 . . . . . . . . . . . . . . . . . . . . . . . . . . . . . . . . . 11

4 Comment évaluer sa consommation d’énergie électrique ? CME2 . . . . . . . . . . . . . . . . . . . . . . . 16

5 Tous les sons sont-ils audibles ? CME3 . . . . . . . . . . . . . . . . . . . . . . . . . . . . . . . . . . . . . . . . . . . . . . . . 21

6 Comment isoler une pièce du bruit ? CME3 . . . . . . . . . . . . . . . . . . . . . . . . . . . . . . . . . . . . . . . . . . . 23

7 Comment éviter le basculement d’un corps ? HS1 . . . . . . . . . . . . . . . . . . . . . . . . . . . . . . . . . . . . . 27

8 Comment soulever un objet ? HS1 . . . . . . . . . . . . . . . . . . . . . . . . . . . . . . . . . . . . . . . . . . . . . . . . . . . 30

9 Comment soulever facilement un objet ? HS1 . . . . . . . . . . . . . . . . . . . . . . . . . . . . . . . . . . . . . . . . . 35

10 Quels sont les composants d’une eau minérale ? HS2 . . . . . . . . . . . . . . . . . . . . . . . . . . . . . . . . . . 38

11 Comment déterminer la concentration d’une substance dans une solution ? HS2 . . . . . . . . . 42

12 Que contient un soda ? HS2 . . . . . . . . . . . . . . . . . . . . . . . . . . . . . . . . . . . . . . . . . . . . . . . . . . . . . . . . . 46

13 Que se passe-t-il lorsque des produits d’usage courant réagissent entre eux ? HS2 . . . . . . . . . . 50

14 Comment préserver son audition ? HS3 . . . . . . . . . . . . . . . . . . . . . . . . . . . . . . . . . . . . . . . . . . . . . . . 54

15 Comment peut-on décrire le mouvement d’un véhicule ? T1 . . . . . . . . . . . . . . . . . . . . . . . . . . . . 57

16 Comment passer de la vitesse des roues à celle de la voiture ? T2 . . . . . . . . . . . . . . . . . . . . . . . . 61

Couverture : npeg .fr

Maquette et mise en page : Nicolas Balbo

© HACHETTE LIVRE 2009, 43, quai de Grenelle, 75905 Paris Cedex 15

ISBN 978-2-01-180538-6

www .hachette-education .com

Tous droits de traduction, de reproduction et d’adaptation réservés pour tous pays.

Le Code de la propriété intellectuelle n’autorisant, aux termes de l’article L . 122-4 et L . 122-5, d’une part, que les « copies

ou reproductions strictement réservées à l’usage privé du copiste et non destinées à une utilisation collective », et, d’autre

part, que « les analyses et courtes citations » dans un but d’exemple et d’illustration, « toute représentation ou reproduction

intégrale ou partielle, faite sans le consentement de l’auteur ou de ses ayants cause, est illicite » .

Cette représentation ou reproduction, par quelque procédé que ce soit, sans autorisation de l’éditeur ou du centre français de

l’exploitation du droit de copie (20, rue des Grands-Augustins, 75006 Paris), constituerait donc une contrefaçon sanctionnée

par les Articles 425 et suivants du Code pénal .

Page 4: COLLECTION DURANDEAU SCIENCES

Le livre du professeur, qui accompagne le manuel de l’élève, comporte les corrigés des activités et des exer-cices proposés dans chaque chapitre .

Les activités dans le manuelNous n’avons pas choisi une méthode déductive fondée sur la présentation d’un cours préalable suivi d’exercices d’application, car le dogmatisme qu’elle peut engendrer nous paraît inadapté dans des clas-ses de Seconde Professionnelle . Nous avons opté au contraire pour une démarche inductive s’appuyant sur des activités, le plus souvent expérimentales, afin de résoudre chaque fois un problème scientifi-que . Cela permet de mieux impliquer l’élève dans la construction de ses connaissances, le professeur l’ac-compagnant, plutôt que le précédant . Le cours (l’es-sentiel) vient ensuite synthétiser les connaissances et apporter les compléments nécessaires sur des sujets qui n’ont pu être abordés dans les activités .Nous avons formulé, dans le manuel, des questions pour chaque activité ; l’élève, en répondant à ces questions, propose des conclusions à la suite de ses observations et mesures . Le professeur peut alors aider à reformuler les résultats qui figurent dans le cours présent dans le manuel .Ces questions jouant un rôle très important dans la démarche, nous proposons les réponses dans le livre du professeur .

Pour aider l’enseignant dans sa tâche :

Le Cédérom met à la disposition de l’enseignant, pour chaque chapitre :- une photothèque contenant les images (sché-mas et photographies) principales du manuel : page d’ouverture, documents sur les activités, le cours, les pages de documentation et les exercices ;- des animations et des vidéos expliquant les notions essentielles, suivies de fiches élève (questionnaires) et de fiches enseignant ;- des schémas bilan animés présentant succincte-ment l’essentiel du cours ;- un exercice différencié à deux niveaux .

Le Livre numérique permet de projeter en per-manence tous les documents nécessaires à la mise en œuvre et à l’exploitation du manuel de l’élève .

Les exercicesDans le livre du professeur, nous fournissons les solu-tions des exercices figurant dans les cinq rubriques du manuel de l’élève .

• Tester ses connaissancesCette forme de présentation et d’évaluation (Q .C .M .) se répand de plus en plus et présente l’avantage de développer l’esprit critique . Cette rubrique permet à chaque élève de tester rapidement les connaissances fondamentales du chapitre .

• Tester ses capacitésCes exercices simples traitent des capacités énoncées dans le référentiel . Chaque concernant une seule capacité, l’élève peut en vérifier plus facilement l ’acquisition .

• Appliquer le coursL’objectif recherché est maintenant plus ambitieux : il s’agit de vérifier l’aptitude des élèves à appliquer les lois du programme dans un contexte proche de celui des activités et de l’essentiel du cours du chapitre .

La résolution de ces exercices nécessite l’acquisition de savoir-faire dans les domaines théorique et expé-rimental .

• Utiliser ses connaissancesLes exercices concernés relèvent d’un niveau de diffi-culté supérieur : il s’agit d’appliquer ses connaissan-ces et ses savoir-faire dans des contextes différents de ceux des activités et de l’essentiel du cours, en parti-culier dans la vie professionnelle . Nous avons intégré dans cette rubrique des exercices dont le thème est le « développement durable » .

• Situation problèmeCes exercices demandent une plus grande autono-mie et davantage de recherche : le travail en groupes se prête particulièrement à la résolution de ces exer-cices .

Le livre du professeur constitue un outil indissociable du manuel, au service des collègues pour le bénéfice de leurs élèves . Nous acceptons bien volontiers leurs suggestions et critiques .

Avant-propos

Page 5: COLLECTION DURANDEAU SCIENCES

4 • CHAPITRE 1 - Quelle différence entre température et chaleur ?

1 Quelle différence entre température et chaleur ? CME1

Les activités

Activité 1 Comment fabriquer un thermomètre pour mesurer la température d’un corps ?

•MatérielUn système de chauffage (chauffe-ballon, ou plaque chauffante, ou bec bunsen)Un ballon de 250 mLUne thermistance CTN reliée à un ohmmètreDes fils de connexionUn thermomètre électronique

•ProduitsEau et glaçons

* Représentation graphique de la température en fonction de la résistance de la thermistance utilisée

10 50 1000

10

50

100Température (°C)

Résistance (Ω)

•Réponseauxquestions

1. Pour la thermistance utilisée : À 0 °C, R(0) = 330 Ω ; à 100 °C, R(100) = 12 Ω .

2. La courbe est la représentation graphique d’une fonction : en effet, à une valeur de la résistance R correspond une seule valeur de la température θ .

3. C’est une CTN, car la résistance diminue lorsque la température augmente (pour une CTP, la résistance augmente avec la température, dans la zone d’utilisation) .

4. On mesure la température d’un corps avec un thermomètre : dans cette activité, on a fabriqué et utilisé un thermomètre électronique dont le capteur est une thermistance CTN .

Page 6: COLLECTION DURANDEAU SCIENCES

CHAPITRE 1 - Quelle différence entre température et chaleur ? • 5

Activité 2 Comment varie la température d’un corps qui échange de la chaleur ?

•MatérielUn système de chauffage (chauffe-ballon, ou plaque chauffante, ou bec bunsen)Un bécher de 100 mLUne thermistance CTN reliée à un ohmmètreDes fils de connexionUn thermomètre électroniqueUn chronomètre

•ProduitsEau et glaçons

Expérience1Lors du chauffage de l’eau, sa température augmente tout au long du chauffage (par exemple de 18 °C à 72 °C) .Lors du refroidissement de l’eau, sa température diminue tout au long du refroidissement (par exemple de 52 °C à 27 °C) . Exemple ci-dessous :

100

10

50Température (°C)

t (min)

Expérience2Tableau des résultats obtenus :

t (min) 0 1 2 3 4 5 6 7 8 9 10 11 12 13

État de l’eau

S : solideL : liquide

S S S S S S S S S S + L S + L S + L S + L L

θ (°C) –1 –1 -0,5 0 0 0 0 0 0 0,5 1 1 1 1,5

•RéponseauxquestionsExpérience1

1. Lorsqu’on chauffe l’eau, sa température augmente . Lorsqu’on la refroidit, sa température diminue .

2. Lorsqu’on chauffe l’eau, le milieu extérieur cède la chaleur et l’eau la reçoit . Lorsqu’on refroidit l’eau, l’eau cède la chaleur au milieu extérieur .

Page 7: COLLECTION DURANDEAU SCIENCES

6 • CHAPITRE 1 - Quelle différence entre température et chaleur ?

3. Un corps froid peut fournir de la chaleur à un autre corps si la température de celui-ci est plus basse que la sienne (c’est le cas dans la seconde expérience) .

Expérience2

4. Lorsque la glace fond, sa température est constante .

5. Le changement d’état observé est la fusion de la glace .

6. La chaleur reçue par la glace provient de l’eau froide .

Les documents

La maison passive

Réponseauxquestions

1. Pour une maison de surface habitable 120 m2, neuve et classique (donc avec une bonne isolation), l’énergie totale nécessaire annuellement est :

110×120 = 13 200 kWh .

2. Dans une VMC « double flux », l’air vicié expulsé de la maison (premier flux) est chaud et il cède de la chaleur à l’air neuf entrant (second flux) qui est à une température plus basse .

Un puits canadien ou provençal est une canalisation contenant un fluide, constituant un réseau soit dans le sol à une certaine profondeur, soit dans une nappe d’eau souterraine : le fluide circulant dans ce réseau reçoit de la chaleur provenant de la terre ou de l’eau et en cède une partie à l’intérieur de la maison .

3. Un pont thermique est une rupture d’isolation thermique entre deux éléments d’un bâtiment dont les résistances thermiques sont différentes .Exemples :- à l’angle entre deux parois verticales ;- entre une paroi verticale et une paroi horizontale ;- au joint entre un mur et un châssis de fenêtre .

La pompe à chaleur géothermique

Réponseauxquestions

1. L’installation géothermique décrite comprend trois circuits de fluides :- le circuit de captage dans le sol ;- le circuit frigorigène de la pompe à chaleur ;- le circuit de chauffage de l’habitation .

2. Le fluide frigorigène récupère la chaleur amenée par le fluide du circuit de captage, à 10 °C environ .

3. Le fluide frigorigène cède la chaleur accumulée au fluide du circuit de chauffage, à 70 °C environ dans notre exemple .

4. Dans l’évaporateur, le fluide frigorigène capte de la chaleur provenant du fluide du circuit de captage et voit sa température augmenter de 0 °C à 5 °C : le fluide frigorigène passe de l’état liquide à l’état gazeux .Dans le condenseur, la température du fluide frigorigène passe de 70 °C à 32 °C : il cède de la chaleur au fluide du circuit de chauffage de l’habitation . Le fluide frigorigène passe de l’état gazeux à l’état liquide .

Page 8: COLLECTION DURANDEAU SCIENCES

CHAPITRE 1 - Quelle différence entre température et chaleur ? • 7

Tester ses connaissancesQ.C.M.1 : B 2 : A et C 3 : C 4 : A 5 : A et B

Tester ses capacités1. Lire un thermomètre1 : 17,5 °C 2 : 68,0 °F 3 : –6 °C 4 : 37 °C .

2. Lire une courbea. Le palier se forme à 42 °C environ : il correspond à un changement d’état du corps .b. Au début de l’expérience, le corps est liquide ; au bout de 9 minutes, il est solide .c. Le changement d’état observé est une solidifica-tion .d. Le corps est de la paraffine, car la température de solidification de l’eau pure est de 0 °C .

Appliquer le cours3. Lire un thermomètre à alcoolTempératures lues : 22 °C ; 21 °C ; 20 °C .La lecture correcte est : 21 °C .

4. Lectures de températuresTempérature mesurées : 39 °C ; –6 °C ; –6 °C .

5. Température de la piècea. Les unités : ° F degré Fahrenheit et °C degré Celsius .b. La température du salon est de 19,5 °C .

6. Changement d’échellea. 0 °C 273 K ; 100 °C 373 K .b. –40 °C 233 K ; 83 °C 356 K .c. 100 K –173 °C ; 293 K 20 °C .

7. Fahrenheita. À un écart de 100 °C correspond un écart de 180 °F ; à un écart de 1 °C correspond un écart de 1,8 °F .b. À la température d’un appartement de 20 °C cor-respond :

32 + 20 × 1,8 = 68 °F .c. À la température du corps humain à 98,6 °F cor-respond : 98,6 – 32 = 66,6 .66,6 ÷ 1,8 = 37 donc 37 °C .

8. Courbes de refroidissementa. La courbe 2 est celle de l’eau pure, car elle possède un « palier » à 0 °C .

b. La température de solidification de l’eau pure est de 0 °C .

c. L’eau salée ne se solidifie pas complètement à une température constante (la courbe ne montre pas de palier) .

9. Expérience

Expérience 1 : on a réchauffé le corps ; le palier observé correspond à la vaporisation de l’eau .

Le corps est bien de l’eau pure .

Expérience 2 : on a refroidi le corps ; le palier observé correspond à une solidification .

Le corps est de la paraffine .

10. Grandeurs et unités

Nom de la

grandeur

Lettre

représentant

la grandeur

Unité de la

grandeur

Symbole

de

l’unité

Quantité de

chaleurQ joule J

Masse du

corpsm kilogramme kg

Capacité

thermique

massique

c

joule par

kilogramme

par degré

Celsius

J/(kg .°C)

Température

initialeθi degré Celsius °C

Température

finaleθf degré Celsius °C

11. Expérience

a. L’eau du verre B reçoit de la chaleur : sa tempéra-ture augmente .

b. L’eau du verre A cède de la chaleur au glaçon : sa température diminue .

12. Bouilloire électrique

a. Quantité de chaleur reçue par l’eau de la bouilloire :

Q = m .c (θf – θi) = 0,5 × 4 180 × (80 – 20) = 125 400 J .

b. La chaleur reçue par l’eau provient de la résistance électrique de la bouilloire .

13. Changement d’état et chaleur

a. Il échange aussi 335 kJ : cette chaleur est fournie par l’eau au milieu extérieur .

b. Lorsque 200 g d’eau se vaporisent à 100 °C, il faut leur fournir : 2 × 226 = 452 kJ .

Les exercices

Page 9: COLLECTION DURANDEAU SCIENCES

8 • CHAPITRE 1 - Quelle différence entre température et chaleur ?

Utiliser ses connaissances14. Chaleur latente

Changement

d’état

Chaleur

latente

massique L

Masse Q (kJ) Chaleur

cédée ou

chaleur

reçue

Fusion 335 kJ/kg 1 kg 335 kJ Chaleur

reçue

Solidification –335 kJ/kg 200 kg –67 000

kJ

Chaleur

cédée

Vaporisation 2 260 kJ/kg 1 kg 2 260 kJ Chaleur

reçue

Liquéfaction –2 260 kJ/kg 500 g –1 130 kJ Chaleur

cédée

15. Choisir un thermomètrea. En France, on utilisera un thermomètre à alcool, car [– 35 °C ; 42 °C] est inclus dans l’intervalle[-117 °C ; 78 °C] . b. Pour repérer la température d’ébullition de l’eau pure (100 °C), on utilisera un thermomètre électroni-que, car 100 appartient à l’intervalle [0 ; 200] .

16. Maison passive (Développement durable)a. La maison passive réalise l’essentiel de ses écono-mies dans le chauffage de la maison .b. La consommation d’énergie pour chaque type de maison :

Type Énergie pour une surface habitable de

110 m2

Maison existante 270×110 = 29 700 kWh

Nouvelle construction 170×110 = 18 700 kWh

Maison basse énergie 110×110 = 12 100 kWh

Maison passive 45×110 = 4 950 kWh

17. La fonte d’un névé (Développement durable)a. Quantité de chaleur nécessaire à la fonte du névé :

Q = m.L = 30 000 × 335 000 = 1 × 1010 Jb. En 1 heure, le névé reçoit :130 × 40 × 1 = 5 200 Wh soit 5,2 kWh ou 5,2 × 3,6 × 106 = 1,87 × 107 J .D’où la durée de la fonte : 1 × 1 010 ÷ 1,87 × 107 ≈ 535 heures soit 540 h à la dizaine d’heures près .

18. Fabrication de potences VTTa. Pour le porter à sa température de fusion :

Q = m.c (θf – θi) = 0,4 × 900 ×(660 – 25) = 228 600 J .

b. Pour fondre 0,4 kilogramme d’aluminium à la température de 660 °C :

Q = m.L = 0,4 × 404 000 = 161 600 J .

c. Lors de sa fusion, la température de l’aluminium reste constante .

19. Je recherche sur le NetAnders Celsius (1701-1744) : professeur d’astronomie à Uppsala en Suède, il construit un thermomètre pour ses recherches en météorologie . Il étudie les aurores boréales, les caractéristiques de la Terre,… .

Daniel Gabriel Fahrenheit (1686-1736) : physicien allemand inventeur de l’aréomètre (permet de mesu-rer des densités) et du thermomètre à mercure qui portent son nom .

a. Celsius avait fixé la première échelle : 0 : tempé-rature d’ébullition de l’eau et 100 : température de congélation de la glace : Il exprimait ses mesures en degré centigrade .

Son échelle a donc été inversée .

b. Fahrenheit a fixé la graduation 0 de son échelle à la température la plus basse dans sa ville de Dantzig lors de l’hiver 1708/1709 .

La graduation 96 correspondait à la température du sang d’un cheval .

20. Eau fraîche en Afrique (Développement durable)La pellicule d’eau à la surface de la jarre se vaporise : le milieu extérieur à cette pellicule doit donc lui four-nir de la chaleur . En particulier l’eau de la jarre qui voit sa température baisser en fournissant de la cha-leur à la pellicule d’eau . Ainsi, l’eau contenue dans la jarre reste fraîche .

21. Solidification d’un corps pura. Première étape : faire fondre de la paraffine dans un tube à essai .

Seconde étape : plonger le tube de paraffine dans un bécher d’eau + glace .

b. Mesurer la température de l’eau dans le bécher .

On observe le changement d’état de la paraffine .

On compare les températures initiale et finale (quand la paraffine est à l’état solide) de l’eau dans le bécher .

L’élévation de cette température montre que la paraf-fine a cédé de la chaleur au milieu extérieur lors de sa solidification .

Page 10: COLLECTION DURANDEAU SCIENCES

CHAPITRE 2 - Quels courants électriques dans la maison ou l’entreprise ? • 9

2 Quels courants électriques dans la maison ou l’entreprise ? CME2

Les activités

Activité 1 Les tensions à la sortie de l’adaptateur d’un téléphone portable et de la prise de courant du secteur sont-elles semblables ?

•MatérielUn ordinateur + interface EXAO ou un oscilloscopeUn adaptateur de téléphone portableUn générateur de courant alternatif 6V/12VDes fils de connexion

•Expérience2Les deux courbes obtenues ressemblent à celles du doc 2 .Dans le cas d’un chargeur de téléphone portable, on peut aussi enregistrer une tension totalement redressée mais non lissée .

•Réponseauxquestions

1. Aux bornes du générateur, la tension observée est alternative (sinusoïdale) donc variable au cours du temps .Aux bornes de l’adaptateur, la tension observée est constante .

2. Le courant du secteur est alternatif ; il est différent du courant continu fourni par un adaptateur de téléphone portable .

Activité 2 Quelles sont les caractéristiques d’une tension alternative ?

•MatérielUn ordinateur + interface EXAO ou un oscilloscopeUn multimètreUn générateur très basse fréquence GTBFUn fréquencemètre (pas obligatoire)Des fils de connexion

•ExpérienceLa courbe analysée ressemble à celle du doc 4 .

•Réponseauxquestions:

1. Dans notre exemple, on dénombre 5 motifs élémentaires, donc 5 périodes en une seconde .La fréquence était réglée exactement sur 5 hertz .Le nombre de périodes par seconde est donc égal à la fréquence .

2. Formule : f = 1

T avec T en seconde et f en hertz .

3. Dans notre exemple, numériquement, Umax = 10 V ; U = 7 V .

On vérifie que :

Umax

U= 2 et

Umax

2=U .

Page 11: COLLECTION DURANDEAU SCIENCES

10 • CHAPITRE 2 - Quels courants électriques dans la maison ou l’entreprise ?

Tester ses connaissancesQ.C.M.1 : A et C 2 : A et C 3 : A, B et C 4 : B et C5 : B 6 : A et B 7 : B et C 8 : B et C .

Tester ses capacités1. Corrigera. Une tension alternative est tantôt positive, tantôt négative .b. Une tension continue est constante au cours du temps .c. Un ordinateur muni d’une interface EXAO (ou un oscilloscope) permet de visualiser les variations d’une tension au cours du temps .

2. ObserverTension continue : b ; Tension alternative : a, c et d .

3. Questionnairea. Une tension est alternative si sa valeur change de signe .b. Une tension est périodique si elle se reproduit régulièrement de manière identique .

4. Analyse de tensionsa. Tension alternative b et d . b. Tension périodique b, c et d .c. Tension alternative périodique b et d .

5. Tension maximale et périodea. La tension maximale est de : 3 × 0,5 = 1,5 V .b. La période est de : 6 × 5 = 30 ms .

6. Motif élémentaire et périodea. Périodes : premier graphe : [ac], deuxième gra-phe : [ab], troisième graphe : [ae] .b. Valeurs : ac = 10 ms ; ab = 50 ms ; ae = 4 s .

7. Lire une périodeT = 8 s ; T = 20 ms .

8. Lire une amplitudea. Amplitudes : 8 V ; 30 V .b. On la nomme aussi « tension maximale » .

9. La tension du secteur

a. U Umax .= = × ≈2 230 2 325 V .

b. Tf

= = =1 150

0 02, s ou 20 ms .

10. Tracé d’une sinusoïde

Période : Tf

= = =1 10 01

100,

s .

Appliquer le cours11. Écran d’EXAOa. Le signal est sinusoïdal .b. T = 20 ms ; f = 50 Hz . C’est la même fréquence que celle du réseau .

c. Umax = 9 V . UU

= = ≈max ,2

9

26 4 V .

12. Prise du secteura. Schéma :

………………………………….....

………………………………….....

………………………………….....

Protection électrique

Phase

Neutre

b. La période du secteur est de 20 ms .c. La valeur maximale de la tension est de 325 V .

13. Mesures et calculsa et b

Symbole Grandeur Appareil de mesure

U Tension efficace Voltmètre

Umax Tension

maximale

Oscilloscope ou

système EXAO

T Période Oscilloscope ou

système EXAO

f Fréquence Fréquencemètre

c. Si f = 50 Hz alors T = 20 ms et non 25 ms .Autre raisonnement : si T = 25 ms alors f = 40 Hz .

14. Lire une façade d’oscilloscopea. Durée de balayage : 5 ms/DIV ; une période occupe 4 DIV d’où T = 4 × 5 = 20 ms .b. Sensibilité verticale : 5 V/DIV ; la hauteur d’un pic occupe 3 DIV d’où Umax = 3 × 5 = 15 V .

15. Lecture d’écran

a. T = 2 × 2 = 4 ms d’où f = 1

T= 1

0,004= 250 Hz .

b. Umax = 2,5 × 5 = 12,5 V d’où U = 12,5

2≈ 8,8 V .

Utiliser ses connaissances16. Au bureaua. La tension d’entrée est alternative sinusoïdale de tension efficace 240 V au plus .L’intensité est inférieure ou égale à 1,5 A . La plage des fréquences est de 50 Hz à 60 Hz .b. La tension de sortie est continue, de valeur 19 V .

Les exercices

Page 12: COLLECTION DURANDEAU SCIENCES

CHAPITRE 3 - Comment protéger une installation électrique ? • 11

L’intensité fournie à l’ordinateur est de 4,74 A .

Il n’y a pas de fréquence indiquée, car la tension n’est pas périodique .

17. Sur le chantier, à l’ateliera. La perceuse sans fil fonctionne sous tension continue ; cette tension électrique est délivrée par des accumulateurs rechargeables .

b. La perceuse Dremel fonctionne en courant alter-natif ; elle se branche directement sur le secteur .

18. Dans la voiture : le GPS embarquéa. Le GPS de voiture fonctionne sous tension conti-nue de 5 V, le courant qui l’alimente ayant une inten-sité de 1 A .

b. Le cordon permet de le brancher sur l’allume-cigare (12 V), car il accepte une tension continue comprise entre 10 V et 30 V ; il supporte une intensité jusqu’à 1,3 A alors que le fonctionnement du GPS ne nécessite que 1 A .

19. Chez le garagiste

a. A : 0,5 tour moteur ; B : 0,5 tour moteur ; C : 1 tour moteur ; D : 1 tour moteur .

b. Il y a bien une injection par tour moteur (courbe D) .

c. A ce régime, un tour moteur dure :

603000

= 0,02 s .

Donc les périodes précédentes sont de :

A et B : 10 ms ; C et D : 20 ms .

20. Développement durable : les modules photovoltaïques

a. Le générateur fournit un courant continu .

b. L’onduleur convertit le courant continu en cou-rant alternatif .

La batterie accumule l’énergie électrique reçue sous forme d’énergie chimique .

c. Un adaptateur de téléphone portable sera bran-ché sous tension alternative, donc sur l’onduleur .

3 comment protéger une installation électrique ? CME2

Les activités

Activité 1 Quel est le rôle d’un coupe-circuit ?

•MatérielpourunposteUn générateur à courant alternatif 6 V4 lampes 6 V/ 0,6 W4 interrupteursDes fils de connexionUn fusible de 250 mA type F rapide sur son porte fusibleUn ampèremètreUne platine

•Réponsesauxquestions

1. L’intensité du courant principal augmente avec le nombre de lampes sous tension . La loi d’additivité des intensités de courant pour un montage en dérivation justifie ce phénomène .

2. L’ouverture du circuit n’est pas instantanée lorsque l’intensité dépasse 250 mA . Le temps de fusion dépend de la surcharge (il sera plus rapide avec quatre lampes qu’avec trois) .

3. En connectant le point A au point B (bornes de chaque branche dérivée), on crée un court-circuit . Le fusible de l’ampèremètre et/ou celui du porte-fusible fondent pour provoquer l’ouverture du circuit .

4. Un coupe-circuit protège les appareils en intervenant lors des courts-circuits ou des surcharges . Il ouvre automatiquement le circuit lorsque l’intensité du courant dépasse pendant un temps déterminé la valeur indiquée par leur calibre .

Page 13: COLLECTION DURANDEAU SCIENCES

12 • CHAPITRE 3 - Comment protéger une installation électrique ?

Activité 2 À quelle condition un disjoncteur différentiel protège-t-il les personnes ?

•MatérielpourunposteUn générateur 12 VUne maquette (câblée sur le schéma TT) comprenant : - un disjoncteur différentiel ; - un module machine ; - une prise trois bornes ; - un mannequin équipé d’une DEL qui s’allume lorsqu’il est électrisé Des fils de connexion

•Réponsesauxquestions

1. La DEL du mannequin s’allume pour signaler l’électrisation dans l’expérience 1 : la carcasse de la machine n’est pas reliée à la terre .

2. Doc 2 : le courant effectue le parcours : fil de phase – carcasse métallique du lave-linge – mannequin (la DEL éclaire, il est électrisé) – fil de mise à la terre par EDF .Doc 3 : le courant effectue le parcours : fil de phase – carcasse du lave-linge – fil de protection électrique (mise à la terre du lave-linge) – fil de mise à la terre par EDF .Le mannequin n’est pas électrisé .

3. Le conducteur différentiel protège les personnes lorsque les carcasses métalliques des machines sont reliées à la terre par un conducteur de protection électrique .

Les documents

Une installation domestique

•Réponsesauxquestions

1. - Le circuit des prises comporte trop de prises .- Même remarque pour le circuit des lampes . De plus, le calibre du fusible est trop faible .- Le chauffe-eau, le congélateur, la cuisinière électrique doivent avoir un circuit par appareil .

2. La nouvelle installation à usage similaire :

NPh

PE

Disjoncteurde 30 mA

Disjoncteur debranchementde 500 mA

Chauffe-eau

5prises

5 la

mpe

s

10 A 16 A

5 la

mpe

s

10 A1,5 mm2

10 A1,5 mm2 2,5 mm21,5 mm2

20 A2,5 mm2 1,5 mm2

cuisinère

32 A6 mm2

congélateurradiateur

16 A

5prises

10 A

Les dangers de la tension monophasée du secteur

•Réponsesauxquestions

1. Il y a contact direct lorsqu’une personne entre en contact avec les parties actives des matériels électriques (soit les conducteurs actifs de la phase ou du neutre, soit les pièces conductrices se trouvant sous tension en service normal : vis, barrettes,…) .

Page 14: COLLECTION DURANDEAU SCIENCES

CHAPITRE 3 - Comment protéger une installation électrique ? • 13

Il y a contact indirect lorsqu’une personne entre en contact avec la masse métallique d’un appareil mise accidentellement sous tension à la suite d’un défaut d’isolement .

2. Le contact direct des deux fils ne produira pas de courant de défaut, car le corps est isolé électriquement de la terre par un tapis isolant . Sans mise à la terre, le disjoncteur différentiel ne détecte pas le défaut . Le courant électrisera la personne .

3. Dans le cas 3, l’oiseau n’est pas électrisé, car il n’est pas soumis à une tension de contact (Uc ≈ 0 V) .

Les exercices

Tester ses connaissancesQ.C.M.

1 : A et C 2 : A et C 3 : A, B et C 4 : B et C

Tester ses capacités1. Protéger du matériel- Un circuit d’éclairage comprenant 8 lampes se protège avec un disjoncteur divisionnaire de 16 A ;

- un circuit de 5 prises de courant se protège avec un disjoncteur divisionnaire de 16 A ;

- un circuit de cuisinière électrique se protège avec un disjoncteur divisionnaire de 40 A .

2. Deux appareils de protectiona. Le calibre de 20 A indique que le coupe-circuit est capable d’interrompre le courant en ouvrant auto-matiquement le circuit lorsque celui-ci dépasse 20 A . Cette coupure intervient au bout d’un certain temps .

b. Leur fonction commune est de protéger le maté-riel installé en cas de surintensité (dépassement du courant nominal) ou de court-circuit . Les deux appa-reils les détectent et coupent le circuit .

c. Le fusible intervient par fusion en cas de sur-charge . On doit changer la cartouche avant de remet-tre la ligne sous tension . C’est la différence avec le disjoncteur divisionnaire qui est muni d’un système de déclenchement mécanique provoquant l’ouver-ture du circuit en cas de surcharge . On le réenclenche pour remettre la ligne sous tension .

(Dans les deux cas, la fermeture du circuit ne se fait qu’après avoir réparé le défaut électrique qui est la cause du déclenchement .)

3. MultipriseL’intensité efficace du courant appelé par la multi-prise est :

I = 3 + 5 + 1,25 = 9,25 A .

4. Additivité des intensitésLe tableau complété :

Interrupteurs

ouverts

K2,

K3

K1,

K3

K1,

K2

K3 K2 aucun

Interrupteurs

fermés

K1 K2 K3 K1,

K2

K1

K3

K1,K2,

K3

Intensité du

courant I (A)

0,17 0,26 0,43 0,43 0,60 0,86

Appliquer le cours

5. Défauts électriques

Les défauts électriques peuvent provoquer des sur-tensions, des surintensités, des courts-circuits ou des courants de défaut .

a. Les surtensions augmentent la tension U (volt) aux bornes des appareils (foudre) .

Les surintensités et les courts-circuits augmentent l’intensité I (ampère) du courant dans les conduc-teurs .

Le courant de défaut Id modifie les valeurs des inten-sités des courants dans le conducteur de phase Iph et le conducteur du neutre In .

On a Id = Iph – In ; Id s’exprime en ampère .

b. Exemples : surtension : la foudre ; surintensité : trop d’appareils utilisant des intensités importantes sur une multiprise ; court-circuit : contact de deux conducteurs dénudés accidentellement ; courant de défaut : usure d’une gaine de conducteur de phase au contact d’une carcasse métallique de machine .

6. Appareils de protection

a. - Les parafoudres protègent des surtensions .

- Les fusibles et les disjoncteurs divisionnaires pro-tègent des courts-circuits et des surcharges .

- Les disjoncteurs différentiels associés à une bonne prise de terre reliée aux masses des machines protè-gent des défauts d’isolement .

b. Les fusibles et les disjoncteurs divisionnaires pro-tègent les installations . Les disjoncteurs différentiels associés à une bonne prise de terre protègent les per-sonnes .

Page 15: COLLECTION DURANDEAU SCIENCES

14 • CHAPITRE 3 - Comment protéger une installation électrique ?

7. Ouverture d’un circuita. Un fusible de 16 A fond lorsqu’il est parcouru pen-dant un certain temps (voir la courbe activité 1) par un courant d’intensité supérieure à 16 A .b. Un disjoncteur différentiel de sensibilité 30 mA se déclenche avant que l’intensité d’un courant de défaut atteigne 30 mA . (Sa plage de déclenchement va de 15 mA à 30 mA) .c. Un disjoncteur divisionnaire, calibre 20 A, se déclenche lorsque la valeur de l’intensité efficace du courant dans la ligne atteint 20 A .d. Le disjoncteur général de branchement de calibre 60 A et de sensibilité 500 mA se déclenche :- lorsque l’intensité efficace du courant appelée par l’installation qu’il protège est supérieure à 60 A (elle correspond à la valeur souscrite à l’abonnement) ;- avant que l’intensité d’un courant de défaut n’at-teigne 500 mA (sa plage de déclenchement va de 250 mA à 500 mA) .

8. Tableau électriquea. Seule une personne habilitée peut intervenir sur un tableau alimenté par la tension du secteur .b. Les dénombrer et les identifier sur le tableau ; il y a autant de coupe-circuits que de lignes .c. Les circuits alimentant les appareils sont séparés afin :- de limiter l’intensité efficace dans les conducteurs de ligne ;- de ne pas mettre hors tension toute une installa-tion en cas de défaut électrique sur un appareil .d. Le circuit appelant la plus grande intensité est protégé par le coupe-circuit de plus grand calibre .

9. Une installationa. L’intensité efficace traversant L2 est : I2 = 0,85 – 0,3 – 0,25 = 0,30 A .b. La puissance transférée par chacune des lampes est : P1 = P2 = 240 × 0,30 = 72 W .

10. Une plaque signalétiquea. La plus grande intensité efficace admise par ce bloc prise est :

I =

PU

= 3680230

= 16 A .

b. L’intensité efficace appelée par les appareils lorsqu’ils fonctionnent est :

I = 6,5 + 5,2 + 7,1 = 18,8 A .Cette intensité efficace dépasse l’intensité nominale prévue par la construction ; donc on n’est pas dans la norme .c. Si les trois appareils fonctionnent simultanément (I = 18,8 A), le fusible ne coupera pas le circuit (calibre 20 A) . L’échauffement des conducteurs de la multi-

prise risque de provoquer un coupe-circuit, car elle est prévue pour admettre un courant d’intensité effi-cace inférieure à 16 A .

11. Circuit « prises de courant »a. Le fusible à cartouche de 16 A protège une ligne admettant 8 points d’utilisation avec une section de conducteur en cuivre de 2,5 mm² .

b. Les trois appareils sont des conducteurs ohmi-ques : ils convertissent l’énergie électrique en cha-leur .

c. La puissance transférée à ces trois appareils est :

P = 600 + 800 + 1 500 = 2 900 W .

Lorsqu’ils fonctionnent ensemble, l’intensité du cou-rant dans le circuit principal est :

I =

2 900230

= 12,6 A .

Le fusible de 16 A n’ouvrira pas le circuit : il protège le circuit .

12. Risque d’électrisation

Contact direct Contact direct

Contact indirect

13. Un enrouleur de rallongea. La section de chaque conducteur est de 1,5 mm² .

b. Lorsque le câble est enroulé, la puissance nomi-nale que peut transmettre l’enrouleur est de 1 200 W . Lorsque le fil est déroulé, cette puissance est de 3 500 W .

c. P = U.I donc I =

PU

= 3 500230

= 15,2 A .

d. Lorsque le câble est déroulé, on peut utiliser cette rallonge, mais le fusible de 10 A ne supportera pas l’intensité efficace de la charge, car 10 A < 15,2 A : le fusible fond .

14. Bonne ou mauvaise prise de terre ?

a. Ra = U

IL

Δ=

× −25

30 10 3 = 830 Ω (Le disjoncteur est de

haute sensibilité) .

Page 16: COLLECTION DURANDEAU SCIENCES

CHAPITRE 3 - Comment protéger une installation électrique ? • 15

b. Si le local est sec, on a UL = 50 V, avec I Δ = 500 mA

la valeur de la prise de terre doit être inférieure à :

500,5

= 100 Ω .

Une prise de terre de 830 Ω constitue un défaut élec-trique, car sa résistance est supérieure à 100 Ω (perte d’efficacité) .

15. Défaut de mise à la terrea. Le tableau complété :

Sensibilité du

différentiel

Résistance

mesurée

Ra (Ω)

Résistance maximale

Rmax = U

IL

Δ

(Ω)

3 A 15 16,7

1 A 40 50

500 mA 150 100

100 mA 400 500

30 mA 550 1 700

b. La prise de terre défectueuse est celle qui est mesurée par un disjoncteur différentiel de 500 mA, car elle est supérieure à 100 Ω(150 > 100) .

Utiliser ses connaissances16. Un conducteur défectueuxa. Le régime de neutre à la terre est dit schéma TT .- Le neutre est directement relié à la terre au niveau du transformateur de distribution de l’énergie élec-trique : premier T .- Les masses d’utilisation sont reliées à une prise de terre par un conducteur de protection électrique : deuxième TLa couleur du conducteur de protection électrique (PE) est jaune et vert .30 mA : le disjoncteur différentiel ouvre le circuit obligatoirement dès que la différence des intensités des courants dans le conducteur de phase et celui du neutre atteint cette valeur .b. Voir schéma : la boucle de courant est surlignée en noir .

Réseau dedistribution

Disjoncteurdivisionnaire

Rn = 25 Ω Ra = 100 Ω

c. Calcul du courant de défaut Id :

Id = U

R Ra n+= 230

125 = 1,84 A .

Avec une sensibilité de 30 mA, le disjoncteur diffé-rentiel détecte le défaut et déclenche l’ouverture du circuit avant qu’une personne entre en contact avec la carcasse de la machine .d. Avant de réenclencher le disjoncteur, il faut effec-tuer les opérations de maintenance nécessaires pour corriger le défaut .

17. Défaut d’isolement et intensité du couranta. Une personne parcourue par un courant de 0,2 mA durant 2 secondes ne court pas de risque .b. Une personne parcourue par un courant de 0,2 A durant 2 secondes risque de graves brûlures, la fibrillation ventriculaire et l’arrêt cardiaque .c. La fibrillation ventriculaire correspond à une série de contractions violentes et désordonnées du muscle cardiaque pouvant entraîner la mort par électrocu-tion .d. Pour éviter le risque de fibrillation ventriculaire, un courant de 100 mA ne doit pas traverser le corps humain durant plus d’une seconde .

18. Résistance du corps humain et tension de contacta. Lorsque la tension de contact augmente, la résis-tance électrique du corps humain diminue .b. Lorsque l’humidité augmente, la résistance élec-trique du corps humain diminue .c. Soumise à 250 V, une peau mouillée a une résis-tance de 800 Ω .d. Uc = Rc .I . L’intensité du courant est I =

250800

= 0,31 A .

e. Il faut isoler ce circuit en moins de 200 ms pour éviter l’arrêt cardiaque .

19. Installer une cuisinière électriquea. La puissance transférée lorsque toutes les plaques fonctionnent est de 7 000 W et le courant appelé est

de

7000230

= 30,5 A . Le fusible fond et ouvre le circuit .

b. Pour les plaques de cuisson, les conducteurs de la ligne d’alimentation doivent avoir une section de 6 mm² et posséder un coupe-circuit de 32 A ou 40 A . Axel et Julianne doivent tirer une nouvelle ligne ayant ces caractéristiques .

Page 17: COLLECTION DURANDEAU SCIENCES

16 • CHAPITRE 4 - Comment évaluer sa consommation d’énergie électrique ?

4 comment évaluer sa consommation d’énergie électrique ? CME2

Les activités

Activité 1 Comment comparer les consommations d’énergie de deux lampes ?

•MatérielDeux lampes montées sur un support : une à incandescence (12 V ; 25 W) et l’autre fluocompacte (12 V – 7 W)Un générateur (12 V)Un joulemètreDes fils de connexion

•RemarqueOn trouve dans le commerce deux types de lampes pouvant fonctionner sous 12 V, ce qui permet une manipulation sous très basse tension en toute sécurité .

•Réponsesauxquestions

1. Pour un éclairage identique, c’est la lampe fluocompacte qui mérite la qualification de LBC, car elle consomme beaucoup moins qu’une lampe à incandescence .

2. Si

E1

E2

= 3,5, on peut écrire que la lampe fluocompacte consomme 3,5 fois moins d’énergie

électrique que la lampe à incandescence .

3. On a

E1

E2

≈P1

P2

≈ 3,5 . L’énergie consommée est donc proportionnelle à la puissance de la lampe .

4. L’utilisation de lampes fluocompactes permet d’économiser de l’énergie électrique .

Activité 2 Quelles sont les grandeurs influant sur l’énergie électrique transférée à un appareil ?

•MatérielUn joulemètre Un générateur Une lampe 6V – 6 W ou autre, d’une puissance de quelques watts Un interrupteur Des fils de connexion

•Réponsesauxquestions

1. L’énergie indiquée par le joulemètre est proportionnelle à la durée de fonctionnement de la lampe .

2. On a P.t ≈ E .

3. L’énergie électrique s’exprime en joule lorsque la puissance P est en watt et la durée de fonctionnement en seconde .

Page 18: COLLECTION DURANDEAU SCIENCES

CHAPITRE 4 - Comment évaluer sa consommation d’énergie électrique ? • 17

Les documents

Différents moyens d’éclairage

•RéponsesauxquestionsLa case cochée indique que la caractéristique est vérifiée :

Caractéristiques

Lampes à

Dure le plus longtemps

Est une lampebasse

consommation

Restitue la plus grande quantité de

lumière par watt

Rend les couleurs correctement

incandescence

fluorescence

DEL

Étude comparative de deux installations d’éclairage

•Réponsesauxquestions

1. Le coût annuel de la consommation est de 39 € (vérification : 150 × 2 × 1 000 × 10-3 × 0,13 = 39) pour l’installation réalisée avec les lampes halogènes et de 8,97 € (vérification : 23 × 3 × 1 000 × 10-3 × 0,13 = 8,97) pour l’installation réalisée avec les lampes fluocompactes .

2. L’installation réalisée avec les lampes fluocompactes présente un surcoût initial de 25 € . La différence des coûts annuels de consommation est de : 39 – 8,97 = 30,03 € . Le surcoût initial est donc amorti au bout de la première année de fonctionnement (25 < 30,03) .

3. L’installation la plus économique au bout de cinq ans est celle des lampes fluocompactes ; l’économie réalisée pendant cette durée est de : 210 – 74,85 = 135,15 soit 135,15 € .

Tester ses connaissancesQ.C.M.1 : B et C 2 : B et C 3 : C 4 : B et C .

Tester ses capacités1. Compteura. La grandeur physique indiquée par l’afficheur est l’énergie électrique consommée .b. Le kilowattheure est une unité pratique .

2. Mesurer une énergie électriqueQ.C.M.1 : A et C 2 : B 3 : C .

3. Énergie pour un cycleb. L’énergie électrique transférée durant ce cycle est 87 547 – 87 532 = 15 kWh .c. Par le calcul, on obtient :

E = P.t = 10 × 1,5 = 15 kWh .

4. Joulemètrea. Le schéma du montage (page 47 doc 2 du livre de l’élève)

U tWeP WI

J

kJA

U V

I

Joulemètre

départmodearrêt

K

G

000000

b. À à la fin de l’expérience, le joulemètre indique :

E = P.t = 12 × 200 = 2 400 J soit 2,4 kJ .

Appliquer le cours5. Conversion et préfixes

a. En utilisant les préfixes,

- en joule : 36 kJ = 3,6 × 104 J ; 72 MJ = 7,2 × 107 J ;

144 TJ = 1,44 × 1014 J .

- en watt-heure : 5 mWh = 5 × 10-3 Wh ;

8 kWh = 8 × 103 Wh ; 9,2 GWh = 9,2 × 109 Wh .

Les exercices

Page 19: COLLECTION DURANDEAU SCIENCES

18 • CHAPITRE 4 - Comment évaluer sa consommation d’énergie électrique ?

b. En utilisant les unités pratiques,- en joule : 3,2 × 103 Wh = 1,152 × 107 J ; 5,2 kWh

= 1,872 × 107 J .- en watt-heure : 36 MJ = 104 Wh ; 54 000 kJ

= 1,5 × 104 Wh .

6. Entre deux relevésa. L’énergie distribuée au cours de cette durée est de 1028 kWh .b. La durée de la consommation a été de 185 jours . (19+30+31+30+31+31+13 = 185) .c. La consommation journalière moyenne d’énergie électrique a été de 5,56 kWh/jour .d. Le montant de cette consommation journalière est de : 5,56 × 0,13 = 0,72 € .

7. Lire avec un joulemètre-wattmètrea.

Valeurs 11,9 V 1,76 A 100 s 20,8 W

Grandeurs

physiques

tension intensité durée puissance

b. L’afficheur indique en kJ : 2,08 .

8. Avec les unités légales

Appareils lampe radiateur Mini

perceuse

E : énergie (J) 7,2 ×104 2,25 ×106 1 920

P : puissance

(W)

40 2 500 16

t : durée (s) 1 800 900 120

9. Avec les unités pratiques

Appareils Réacteur

nucléaire

Calculatrice Lave-linge

E : énergie 216 GWh 10,8 J 3 kWh

P : puissance 900 MW 0,06 W 2 000 W

t : durée 10 jours 3 minutes 1,5 heure

10. Coût d’un oublia. Énergie perdue : E = 30 × 24 × 0,1 = 72 kWh .b. Le coût est de : 0,13 × 72 = 9,36 € .

11. Deux cycles d’un lave-vaissellea. La puissance électrique moyenne consommée lors du cycle de lavage est :

P =

7000,5

= 1 400 W .

b. L’énergie électrique consommée par le lave-vais-selle lors du cycle de lavage est :

875 × 2 = 1 750 Wh ou 1,75 kWh .c. Le coût du cycle rapide est de : 0,70 × 0,13 = 0,09 € .Celui du cycle intensif est de : 1,75 × 0,13 = 0,23 € .La différence de coût entre ces deux cycles est de 0,14 € .

12. Budget TVa. Le téléviseur a fonctionné pendant t =

EP

= 0,520,13

= 4 h .b. L’énergie consommée par le téléviseur de Jules en un an est : E = 0,52 × 365 ≈ 190 kWh .c. Le coût de cette consommation d’énergie est de : 0,13 × 190 = 24,70 € .

13. Ordinateur portablea. Représentation

00 1 2 3 4

10

20

30

40

50

60E (Wh)

5

t (h)

b. Les grandeurs E et t sont proportionnelles, car la droite représentative de E = f(t) passe par l’origine .c. Le coefficient permettant de calculer E (Wh) lorsqu’on connaît la durée t (h) est la puissance P de l’ordinateur . Ici, P = 12 W .d. L’énergie électrique transférée par l’ordinateur pour un fonctionnement mensuel de 60 h est :

E = P.t = 12 × 60 = 720 Wh,soit 50 fois moins qu’un ordinateur en forme de tour .

14. Lampadaire halogène (développement durable)a. La puissance transférée par des lampes LBC pour obtenir la même quantité de lumière est de 60 W .b. Il faudra 3 lampes LBC de 20 W pour remplacer le lampadaire halogène .

Utiliser ses connaissances15. Maîtriser l’énergie (développement durable)a. Cet appareil est de la classe A .b. Les machines les plus « énergivores » sont de la classe G .c. Durant un cycle de fonctionnement, une énergie de 1,4 kWh est consommée .d. Coût d’un cycle de lavage : 1,4 × 0,13 = 0,18 € .

16. La petite boulangeriea. Dans la première pièce, la puissance transférée est

P1 = 3 000 + 1 000 + 4 × 75 = 4 300 W .Le produit : U.I = 230 × 22 = 5 050 W .La puissance transférée est inférieure à U.I, car les machines fonctionnent avec des moteurs donc la for-mule P = U.I n’est pas applicable et l’on a P < U.I .

Page 20: COLLECTION DURANDEAU SCIENCES

CHAPITRE 4 - Comment évaluer sa consommation d’énergie électrique ? • 19

b. La puissance consommée dans la deuxième pièce lorsque tous les appareils fonctionnent est

P2 = 2 400 + 800 + 5 × 60 = 3 500 W .c. Le produit : U. × I = 230 × 15,3 = 3 500 W . On a P = U.I : les appareils sont des dipôles résistifs .Lorsque tous les appareils fonctionnent, la puissance transférée est :

P1 + P2 = 4 300 + 3 500 = 7 800 W .La puissance souscrite est de 9 000 W, donc ils peu-vent fonctionner simultanément .

17. Petit déjeunera. Énergie consommée : E = P .t = 800 × 5 × 60 = 2,4 × 105 J .b. Intensité efficace : I =

PU

= 800230

= 3,48 A .

c. Résistance du grille pain : R = UI

= =2303 48,

66 Ω .

18. Veille voleuse (développement durable)a. L’énergie consommée par jour :- lorsque J . Blackwatt regarde la télévision :

E = P .t = 0,1 × 3 = 0,3 kWh ;- lorsque le poste est en mode veille :

E = P .t = 0,016 × 21 = 0,336 kWh .b. S’il éteint le poste, il fera une économie de : 0,336 × 0,13 = 0,04 € par jour, soit 16 € par an .c. Autres appareils conçus sur le même type d’arrêt : un décodeur, un ordinateur, un lecteur de DVD, une box…

19. Consommation électrique d’une maison (développement durable)a. Les appareils consommant près du tiers de l’éner-gie électrique sont le réfrigérateur et le congélateur .b. Diagramme circulaire à secteur représentant les consommations électriques de la maison :

32 %

14 %14 %

12 %

7 %

14 %

7 % froid

éclairagelave-vaisselle

sèche-linge

audiovisuel

lave-lingeautres

c. Précautions à prendre pour qu’une famille réalise des économies d’énergie électrique :- utiliser correctement les thermostats des appa-reils pour réguler la température ;- utiliser des doubles ou triples vitrages ;

- veiller à éloigner des sources de chaleur les appa-reils créant du froid (réfrigérateur, congélateur) ;

- éteindre chaque fois que cela est possible les appareils en mode veille ;

- utiliser des lampes basse consommation (fluores-centes ou électroluminescentes) ;

- au moment du remplacement des appareils, choi-sir des appareils économisant l’énergie (classe A ; A+) ;

- isoler thermiquement les habitations (voir la réglementation technique en vigueur) .

d. ADEME signifie : Agence De l’Environnement et de la Maîtrise de l’Energie . Cette agence s’occupe des déchets, de la pollution, de l’énergie, des matériaux renouvelables, du développement durable…

20. Sur le Net ou au CDI

Se renseigner sur :

a. James Watt (1736 – 1819) a permis, par ses travaux, l’amélioration de la machine à vapeur qui servait à l’extraction du charbon dans les mines . Le nom de Watt a été donné à l’unité de puissance .

James Joule (1818 – 1889), après des études scientifi-ques à l’Université de Cambridge, reprit l’entreprise familiale de brasseur de bière et y construisit un labo-ratoire de recherche . Ses nombreuses études ont porté sur l’énergie, la chaleur et l’électricité . Il a montré, en particulier, que la chaleur est une forme d’énergie . Le nom de Joule a été donné à l’unité d’énergie .

b. Thomas Alva Edison (1847 – 1931) est reconnu comme l’un des inventeurs américains les plus importants . C’est le fondateur de l’un des plus grands empires industriels mondiaux : Général Electric . Il revendique un nombre record de 1 093 brevets et inventions : le télégraphe ; « transmetteur-receveur duplex automatique de code Morse » ; le phonogra-phe ; l’ampoule électrique ; le kinétographe ; le kiné-toscope Parlors ; l’accumulateur alcalin nickel-fer …

21. Un atelier en hiver

- L’appareil électrique de plus grande puissance est la scie à bande avec 6 kW (Sylvestre Bobois ne peut utiliser qu’une machine à la fois puisqu’il travaille seul . De plus, il doit économiser l’énergie en étei-gnant les machines lorsqu’il ne les utilise pas) .

- Le chauffage et l’éclairage consomment une puis-sance de 4 + 0,2 = 4,2 kW .

- Il utilise donc au maximum 4,2 + 6 = 10,2 kW . Avec un abonnement de 12 kW, il peut donc s’éclairer et se chauffer tout en travaillant en hiver .

Page 21: COLLECTION DURANDEAU SCIENCES

20 • CHAPITRE 4 - Comment évaluer sa consommation d’énergie électrique ?

22. Vérifier sa facture avec un tableur (TIC)

électricité123456789101112

A C D E F G H I JBancien relevé nouveau différence montant HT ()prix kWh () taxes locales TVA total TTC ()

47,87 4,59 6,16 58,62abonnement 4.45 / mois 26,70consommation du 26/10/08 au 25/04/09 10949 11218 0,0787 21,17269 269

montant HT () taxes locales TVA TTC ()autres prestations 0,24 1,451,21

1,210,0045269

montant HT () taxes locales TVA TTC ()total 49,08 4,59 6,40 60,07

consommation (kWh)

23. Geste recyclagea. Les lampes à recycler dans la filière lampes usa-gées sont les lampes fluocompactes, les tubes fluo-rescents et les lampes à DEL (sur leurs emballages figure une poubelle barrée) .

b. Les lampes fluocompactes et les tubes fluores-cents contiennent du mercure . Celui-ci ne doit pas se retrouver dans la chaîne alimentaire à cause de sa haute toxicité .

c. Il faut déposer ces lampes dans un point de col-lecte prévu à cet effet dans les déchetteries ou les magasins qui les vendent . Leur transfert et leur recy-clage sont financés par l’éco-contribution payée lors de l’achat .

24. Recherchera. Le rapport des puissances consommées entre une lampe à incandescence et une lampe basse consom-mation est de 5 .

b. En 1802, Humphry Davy découvre que le passage du courant électrique dans un fil conducteur élève sa température et le fait fondre rapidement . En 1880, Thomas Alva Edison invente la lampe à incandes-cence en plaçant une fibre de bambou du Japon dans une ampoule sous vide . Cette fibre soumise à une faible tension produit de la lumière . Ce filament fut remplacé par la suite par un filament de tungstène . Le passage du courant électrique produit un échauf-fement portant à incandescence le filament . La durée de vie de ce type de lampe est de 1 000 à 2 000 heures .

c. Dans les lampes fluorescentes (aussi appelées tubes ou lampes fluocompactes), des décharges élec-triques fournissent un courant . Les chocs des élec-

trons avec des atomes de mercure produisent des ultraviolets transformés en lumière visible en traver-sant une poudre fluorescente qui recouvre les parois .

atome de mercuregaz neutre (argon)

poudre fluorescente

lumière blanchevisible

poudre fluorescente

Les lampes électroluminescentes (DEL) : la jonc-tion entre deux cristaux semi-conducteurs émet un rayonnement lumineux lorsqu’elle est parcourue par un courant . Enfermées dans une résine, ces lampes résistent aux vibrations . Associées en nombre et com-mandées par l’électronique, les DEL sont devenues à haute luminosité .

d. Les rendus des couleurs sont différents pour les trois lampes . Le meilleur (proche de la lumière blan-che naturelle) est celui de la lampe à incandescence . Les spécialistes utilisent l’indice de rendu des cou-leurs (IRC) pour caractériser la lumière des lampes . Sa valeur maximale 100 correspond à la lumière du jour . Une lampe à incandescence ordinaire à un indice supérieur à 95 alors que l’éclairage fluorescent va de 50 à 80 suivant la composition de la poudre contenue dans la lampe . Les DEL ont un indice médiocre .

Page 22: COLLECTION DURANDEAU SCIENCES

CHAPITRE 5 - Tous les sons sont-ils audibles ? • 21

5 tous les sons sont-ils audibles ? CME3

Les activités

Activité 1 Comment déterminer la fréquence d’un son ? Quelle est la gamme de fréquences d’un son audible

par l’homme ?

•MatérielUn GBFUn haut-parleurUn interrupteurUn microphoneUn oscilloscope

•Réponsesauxquestions

Expérience1

1. Lorsque la fréquence augmente, le son devient plus aigu .

2. Oui, les deux fréquences sont égales .

Expérience2

3. L’intervalle des fréquences des sons audibles est en théorie de [20 Hz ; 20 kHz] . En général, il est plus restreint dans la réalité : une personne âgée n’entend plus les sons au-delà de 10 à 12 kHz .

4. Notre oreille n’est sensible qu’aux sons de certaines fréquences . En dessous de 20 Hz et au dessus de 20 kHz, nous ne percevons pas les vibrations contrairement à certains animaux .

Activité 2 Quelle grandeur se mesure en décibel ?

•MatérielUn GBFUn haut-parleurUn sonomètreUn voltmètreUne boîte d’expérimentation (si possible), dans laquelle on peut fixer un haut-parleur et possédant un trou pour le capteur du sonomètre .

•Réponseauxquestions

1. Non, le niveau d’intensité acoustique du « silence » n’est pas égal à zéro .

2. Lorsque la tension aux bornes d’un haut-parleur augmente, le niveau d’intensité acoustique augmente . Lorsque le son est « plus fort », le niveau d’intensité acoustique augmente .

3. Nous ne percevons que certains sons, car notre oreille n’est sensible qu’à partir d’une valeur minimale de niveau d’intensité sonore (voir le diagramme de Fletcher et Munson) .

Page 23: COLLECTION DURANDEAU SCIENCES

22 • CHAPITRE 5 - Tous les sons sont-ils audibles ?

Tester ses connaissancesQ.C.M1 : C ; 2 : A ; 3 : A et B ; 4 : A ; 5 : C ; 6 : C ; 7 : A, B, C .

Tester ses capacités1. Un son purLe son pur correspond à l’oscillogramme a .Le son complexe correspond à l’oscillogramme b .Le bruit correspond à l’oscillogramme c .

2. Mesure d’une période sur un oscilloscopeLes enregistrements a et b sont périodiques . Pour chacun de ces enregistrements, un motif est dessiné ci-dessous :a. b.

Appliquer le cours3. Mesure d’une période à partir d’un enregistrement EXAOa. Pour l’enregistrement 1, T = 2,23 ms d’où f = 448 Hz .b. Pour l’enregistrement 2, T = 2,25 ms d’où f = 444 Hz .

4. Comment positionner un sonomètre ?a. La bonne position est celle schématisée sur le des-sin 3 .b. Il faut paramétrer le sonomètre pour qu’il mesure en dB(A) .

5. Quels réglages pour quels effets ?

a.

GBF

HP

b. Il faut régler la fréquence de la tension de sortie du GBF .c. Il faut régler la valeur de la tension de sortie du GBF (bouton « Amplitude ») .

6. Mettre de l’ordreLes sons les plus graves sont ceux dont la fréquence

est la plus basse . Le classement des sons du plus grave au plus aigu est donc : son 4 ; son 2 ; son 5 ; son 3 ; son1 .

7. Sans calculLes sons les plus graves sont ceux dont la période est la plus grande . Le classement des sons du plus aigu au plus grave est donc : d ; b ; a ; c .

8. Un seul mota. Pour augmenter la hauteur d’un son, il faut aug-menter sa fréquence . Pour diminuer la hauteur d’un son, il faut diminuer sa fréquence .b. Un son aigu a une fréquence comprise entre 1,5 kHz et 20 kHz .c. Les ultrasons ont une fréquence supérieure à 20 kHz .d. En dessous de 20 Hz, un son n’est pas audible par l’homme .

9. Calcul de fréquencesa. Enregistrement a : Ta = 2 ms ; enregistrement b : Tb = 2 ms ; enregistrement c : Tc = 20 ms ;b. Enregistrement a : fa = 500 Hz ; enregistrement b : fb = 500 Hz ; enregistrement c : fc = 50 Hz .c. Les sons a et b sont plus aigus que le son c .d. La période diminue quand le son devient plus aigu .

10. Fletcher et Munsona. Pas toujours . Par exemple, un son de 50 Hz et 40 dB n’est pas audible .b. Non . Seulement si L > 5 dB .c. Par exemple, un son de (3 000 Hz ; 95 dB) .d. Entre 2500 et 4000 Hz .

11. Quel bouton ?Les réglages sont ceux de la photo .a. C’est une tension alternative sinusoïdale (bouton rouge de droite) .b. La fréquence est donnée à l’écran . Ici f = 400 Hz .c. Le haut-parleur émet un son de fréquence égale à celle de la tension qui l’alimente . Donc ici, la fré-quence du son émis vaut 400 Hz .

Les réglages sont modifiés :d. Non, cela va augmenter ou diminuer le niveau d’intensité sonore, mais pas la fréquence .e. Il faut se placer sur le calibre 100 kHz et tourner le bouton fréquence jusqu’à ce que l’écran affiche 14 000 .f. fmax = 100 kHz . Dans ce cas, le son est inaudible, car f > 20 kHz . C’est un ultrason .g. fmin = 1 Hz . Le son est un infrason .

Les exercices

Page 24: COLLECTION DURANDEAU SCIENCES

CHAPITRE 6 - Comment isoler une pièce du bruit ? • 23

12. Additivité de deux sources sonoresa. « Si deux sources sonores émettent des sons de même niveau d’intensité sonore, le niveau d’inten-sité sonore total augmente de trois . »b. « Si deux sources sonores émettent des sons dont le niveau d’intensité sonore diffère de 10 dB, le niveau d’intensité sonore global est sensiblement égal à la valeur du niveau d’intensité sonore de la source la plus forte . »

13. Quel aspirateur ?a. Daison, car c’est pour celui là que le niveau d’in-tensité acoustique L est le plus petit (74 dB) .b. Non . Ces deux grandeurs ne sont pas liées .

14. Un haut-parleur adaptéa. Non, car un son grave a une fréquence comprise entre 20 Hz et 300 Hz .b. Non, car les fréquences d’une conversation sont comprises entre 250 Hz et 3 500 Hz . Ce haut-parleur ne retransmettra que les sons les plus aigus de la conversation .c. Non, car les ultrasons ont des fréquences supé-rieures à 20 000 Hz .d. Oui, car les douleurs apparaissent environ à 120 dB .

15. Régler la hauteur du son d’un téléviseura. La fréquence d’un son grave est comprise entre 20 Hz et 300 Hz . Quentin doit donc positionner le curseur 100 Hz au maximum .

b. Un son aigu a une fréquence comprise entre 1,5 kHz et 20 kHz . Quentin doit donc positionner les curseurs 3 kHz et 10 kHz au maximum .

16. Arthur accorde sa guitarea. La période de ce signal vaut T = 3 ms . La fréquence est donc égale à :

f = 1 1

3 10333

3T=

×=

−Hz .

b. Il faut qu’Arthur diminue la fréquence du son émis par la corde . Pour cela, il doit la détendre afin d’allonger la partie vibrante de la corde .

17. Jouer de la guitare sèchea. Oui, car les sons médiums ont une fréquence com-prise entre 300 Hz et 1 500 Hz . En plaçant ses doigts sur la corde, Arthur va la raccourcir et donc augmen-ter la fréquence du son émis .b. Non, car la fréquence du son ne peut qu’augmen-ter si on raccourcit la partie vibrante de la corde . Or les sons graves ont une fréquence inférieure à 300 Hz .

18. Situation problèmea. Oui, car la détérioration des capacités auditives commence à environ 80 dB .b. Le responsable doit fournir des dispositifs de pro-tection individuelle contre le bruit et s’assurer que les employés les utilisent .

6 comment isoler une pièce du bruit ? CME3

Les activités

Activité 1 Le double vitrage assure-t-il une meilleure isolation phonique que le simple vitrage ?

•MatérielUn GBF - Un haut-parleur - Un interrupteur - Un sonomètre - Une caisse

Exemple de relevé des niveaux d’intensité acoustique (en décibel) pour différents vitrages

Fréquence (Hz)Air ou vitrage

125 500 1 000 4 000 8 000

Air 82,7 90 90 90 83

Verre 4 mm 59,5 48,1 59 45,9 49,9

Verre 10 mm 53,5 43,2 55,6 41,1 45,2

Double vitrage 4/10/4 55,5 47,5 57,3 43,3 46,8

Double vitrage 10/10/4 50,7 40,2 53,1 39,2 41,2

Page 25: COLLECTION DURANDEAU SCIENCES

24 • CHAPITRE 6 - Comment isoler une pièce du bruit ?

•Réponsesauxquestions

Expérience1

1. Oui, le verre est un isolant acoustique : L(verre) < L(air) .

2. Oui, l’épaisseur du verre joue sur le niveau d’isolement : L(verre 10 mm) < L(verre 4 mm) .

3. Non, chaque matériau a une fréquence de résonance pour laquelle il isole moins bien .

Expérience2

4. Non, un double vitrage 4/10/4 n’isole pas mieux qu’un simple vitrage de 10 mm .

Activité 2 Comment évolue le niveau d’intensité acoustique lorsque le récepteur s’éloigne de la source sonore ?

•MatérielUn GBFUn haut-parleurUn interrupteurUn sonomètreUne caisse

Distance d entre le haut-parleur et le sonomètre (cm) 5 10 20 40

Niveau sonore L en dB (arrondi à l’unité) 80 74 68 62

•Réponsesauxquestions

1.

0

60

65

70

75

80

10 20 4050

d (cm)

L (dB)

2. Le niveau d’intensité acoustique diminue lors que la distance entre la source et le récepteur augmente . Cette diminution est de 6 décibels chaque fois que la distance double .

Page 26: COLLECTION DURANDEAU SCIENCES

CHAPITRE 6 - Comment isoler une pièce du bruit ? • 25

Les exercices

Tester ses connaissancesQ.C.M.1 : C ; 2 : B ; 3 : A et C ; 4 : B ; 5 : B .

Tester ses capacités1. Le meilleur matériauLe classement des matériaux du plus performant au moins performant est : matériau 3 ; matériau 4 ; maté-riau 1 ; matériau 2 .

2. L en fonction de da. Le niveau d’intensité acoustique L diminue lors-que la distance d entre la source sonore et le récep-teur augmente .b.

Distance de mesure

d (m)2 4 16 32

Niveau d’intensité

acoustique L (dB)98 92 80 74

3. Vrai – Faux Le niveau d’intensité acoustique :a. Faux : se note L .b. Faux : se mesure avec un sonomètre .c. Faux : diminue lorsque la distance augmente .d. Vrai : décroît lorsque la distance augmente .e. Faux : augmente lorsque la distance diminue .

Appliquer le cours4. Dans un immeublea. Il doit intervenir pour que l’isolement phonique entre deux appartements soit au moins de 53 dB et que l’isolement entre la cage d’escalier et un appar-tement soit au moins de 53 dB (voir le document sur les niveaux règlementaires d’isolation, page 72 du manuel) :- entre le studio 1 et le studio 2 ;- entre le studio 2 et la cage d’escalier ;- entre le studio 4 et la cage d’escalier .b. Nicolas devrait faire mesurer- le niveau d’isolement des façades ;- le niveau d’isolement des sols et des plafonds aux bruits d’impacts et aux bruits aériens .

5. Épaisseur du vitragea. Le verre isole plus des sons aigus que des sons gra-ves, car les sons aigus ont des fréquences plus élevées que les sons graves .b. Plus le verre est épais, plus l’atténuation phonique est importante .c. L’atténuation dans ce cas est de 40 décibels .

6. Épaisseur d’air (les indices)

a. Dans ce cas Rw ≈ 41,5 dB .

b. Rw augmente d’environ 3,75 dB chaque fois que l’épaisseur du verre double .

7. Isolation par parois doubles

a. Quand on remplace la couche d’air par un maté-riau isolant, on augmente l’indice d’affaiblissement acoustique .

b. Non les deux schémas centraux montrent que deux murs identiques en série n’isolent pas deux fois plus qu’un mur seul .

c. Les deux plaques de plâtre intérieures sont reliées aux plaques de plâtre extérieures par les poteaux . Lorsque le son percute la paroi, l’onde sonore est transmise à la plaque de plâtre intérieure par le poteau . Elle traverse donc la cloison . Si la plaque de plâtre intérieure est enlevée, l’onde sonore n’est plus transmise par ce biais et l’atténuation sonore aug-mente .

8. Loi des masses

a. Les trois paramètres qui influent sur l’isolation acoustique d’une paroi sont : la fréquence du son, l’épaisseur de la paroi, la qualité du matériau utilisé .

b. L’atténuation est meilleure pour les sons aigus, d’après le diagramme .

c. Dans ce cas, l’affaiblissement Rw a une valeur de 28 dB .

d. On peut utiliser de la brique pleine de 5,5 centi-mètres d’épaisseur .

9. Calfeutrage

a. Faux . Le son n’a besoin que de petits espaces pour se propager : « Là où l’air passe, le bruit passe » .

b. Faux . Si l’air passe sous une porte, le bruit passe forcément .

c. Vrai . Des fenêtres étanches à l’air sont indispensa-bles pour améliorer l’isolation phonique .

d. Vrai . Le calfeutrage parfait d’une pièce est bon pour l’isolation mais mauvais pour la santé . Une pièce saine doit être aérée .

10. Capotage insonorisant

a. Les principes évoqués sont :

- la loi de masse : plus le poids au m² d’une cloison est important, plus elle sera efficace pour isoler des sons ;

- la loi d’étanchéité : une paroi doit être parfaite-ment étanche pour isoler du bruit .

b. Le plomb est le meilleur isolant .

c. Oui, une petite fuite peut limiter grandement l’isolement du capot .

Page 27: COLLECTION DURANDEAU SCIENCES

26 • CHAPITRE 6 - Comment isoler une pièce du bruit ?

Utiliser ses connaissances11. Connaître et respecter les limitesa. dB signifie décibel . C’est l’unité du niveau d’inten-sité sonore L .b. Non, le niveau d’intensité sonore au calme à la campagne est d’environ 30 dB .c. Le seuil de fatigue auditive est de 85 dB .d. Dans un bureau : L ≤ 40 dB .e. Dans un atelier d’ajustage : 8 h maximum . Dans une chaudronnerie : 2 minutes maximum .f. Oui . (L = 105 dB .)g. Test de moteur sur un banc d’essai .h. Écouter un baladeur, aller en discothèque, suivre un concert de rock, chasser .

12. Rénover son appartementa. En 2 : permuter RW et LW : Rw est un indice d’af-faiblissement des bruits aériens et Lw un indice d’af-faiblissement aux bruits d’impacts . Il ne faut pas confondre les indices de performance des matériaux avec les performances de la paroi .b. Afin d’assurer l’étanchéité des ouvertures et limi-ter le passage de l’onde sonore .c. Il pourrait isoler les planchers et les plafonds, remplacer les vitrages par des vitrages spécifiques .

13. Trafic aérien.a. À 300 mètres, L ≈ 103 dB .b. À 18 m, L18 ≈ 127 dB . À 1 200 m, L1200 ≈ 91 dB . À 4,8 km, L4800 = 79 dB .

Méthode de calcul destinée au professeurL’intensité acoustique I est inversement proportion-nelle au carré de la distance d :

I = k/d2 où k est une constante .Le niveau d’intensité acoustique est donné par :

L = 10 log(I/I0) avec I0 = 10–12 W .m–2 .

L = 10 log k

d I20 .

= 10 log (

kI0

) – 20 log d = K – 20 log d

où K est une autre constante .Pour la distance d1, L1 = K – 20 log d1 .Pour la distance d2, L2 = K – 20 log d2 .

L2 – L1 = 20 log (d1/d2) .L’abaque donne L1 = 103 dB pour d1 = 300 m, lors du décollage .Calculons L2 pour d2 = 18 m :

L2 – L1 = 20 log (300/18) = 20 log 16,7 = 24 .Donc L2 = 127 dB .Calculons L3 pour d3 = 1 200 m :L3 – L1 = 20 log (300/1 200) = –20 log 4

= –20 log 22 = –40 log 2 = –40 × 0,3 = –12 .Donc L3 = 91 dB .Calculons L4 pour d4 = 4 800 m :L4 – L1 = 20 log (300/4 800) = –20 log 16 = –20 log 24

= –80 log 2 = –24 .

Donc L4 = 79 dB .

Méthode de calcul adaptée à l’élèveOn procède par itération sachant que le niveau dimi-nue de 6 dB quand la distance double .

- Évaluation du niveau d’intensité acoustique L2 à 18 m, connaissant le niveau L1 = 103 dB à 300 m :

Distance (en m) Niveau d’intensité acoustique (dB)

18 L2

36 L2 – 6

72 L2 – 12

144 L2 – 18

288 L2 – 24

La distance 288 m est voisine de 300 m et le niveau d’intensité acoustique est voisin de L1 = 103 dB .

Donc L2 – 24 ≈ 103, soit L2 ≈ 127 dB .

NB : Ce résultat constitue une bonne approximation, car la fonction logarithmique décroît lentement avec la distance .

- Évaluation du niveau d’intensité acoustique L3 à 1200 m, connaissant le niveau L1 = 103 dB à 300 m :

Distance (en m) Niveau d’intensité acoustique (dB)

300 L1

600 L1 – 6

1 200 L1 – 12

L3 = L1 – 12 = 103 – 12 = 91 dB .

- Évaluation du niveau d’intensité acoustique L4 à 4 800 m, connaissant le niveau L1 = 103 dB à 300 m :

Distance (en m) Niveau d’intensité acoustique (dB)

300 L1

600 L1 – 6

1 200 L1 – 12

2 400 L1 – 18

4 800 L1 – 24

L4 = L1 – 24 = 103 – 24 = 79 dB .

c. Isolement = 79 – 35 = 44 dB . L’isolement d’une chambre située à 4,8 kilomètres doit être au moins égal à 44 décibels .

14. Cloisons d’isolation phoniquea. La cloison F530 offre les meilleures performances acoustiques (courbe rouge) .

b. L’indice de la cloison 72/48 chute pour les fré-quences supérieures à 2000 Hz .

c. Oui, car une partie seulement de la bande de fré-quences des sons des conversations sera mal atté-nuée .

d. L’isolation permet une isolation de 40 à 70 déci-bels selon les fréquences . Cette cloison permet donc une isolation convenable .

e. Non, car il ne faut pas confondre l’indice d’isole-ment des matériaux et l’isolement du mur .

Page 28: COLLECTION DURANDEAU SCIENCES

CHAPITRE 7 - Comment éviter le basculement d’un corps ? • 27

7 comment éviter le basculement d’un corps ? HS1

Les activités

Activité 1 À quelle condition un objet peut-il basculer ?

•MatérielUn parallélogramme déformable muni d’un fil à plomb

Expérience- Présenter le parallélogramme déformable, son centre de gravité G ainsi que la verticale de G matérialisée par le fil à plomb . Présenter la surface d’appui sur le plan, appelée base de sustentation .- Déformer progressivement le parallélogramme jusqu’à ce qu’il se renverse .

•Réponsesauxquestions

1. Le parallélogramme reste en équilibre sur le plan horizontal tant que la verticale du centre de gravité, matérialisée par le fil à plomb, rencontre la base de sustentation .

2. Le parallélogramme bascule dès que la verticale du centre de gravité, tombe en dehors de la base de sustentation .

3. Lorsque le camion a déployé la grue, le centre de gravité du système camion-grue s’est déplacé et sa verticale est sortie de la base de sustentation comprise entre les quatre roues . Le camion s’est alors renversé . Le camionneur aurait dû sortir les stabilisateurs pour augmenter l’aire de la base de sustentation .

Activité 2 Quelles sont les caractéristiques du poids d’un corps ?

•MatérielUn support, une noix, une tigeUn fil à plombUne boulette de pâte à modelerUne boîte d’allumettesUn dynamomètreUne balance électronique

Expérience- Matérialiser la direction du fil à plomb par un petit morceau de pâte à modeler . Remonter le fil et le brûler . Vérifier que le point d’impact coïncide avec la trace marquée sur la pâte à modeler .- Mesurer le poids du fil à plomb avec un dynamomètre .- Mesurer sa masse avec une balance électronique .

•Répondreauxquestions

Expérience1

1. Le poids du fil à plomb s’applique au centre de gravité .

2. Le fil à plomb tombe suivant la direction verticale .

3. Le sens de la chute est du haut vers le bas .

4. La valeur du poids du fil à plomb est P = 1,6 N .

Expérience2

5. La valeur de la masse du fil à plomb est m = 0,159 kg .

6. La valeur du rapport

Pm

=

1,60,159

= 10 N/kg .

Page 29: COLLECTION DURANDEAU SCIENCES

28 • CHAPITRE 7 - Comment éviter le basculement d’un corps ?

Les documents

Les plaques de charge sur les chariots frontaux

•Répondreauxquestions

1. La hauteur de gerbage est de 3,20 m, inférieure à 3,30 m . C’est donc la courbe 1 qu’il faut utiliser . Le centre de gravité G est situé à 600 mm du mât . L’ordonnée correspondant à l’abscisse 600 mm sur la courbe 1 est 1 800 kg . La charge maximale mesurant 1,2 m de côté pouvant être gerbée à 3,20 m est 1 800 kg .

2. La hauteur de gerbage est maintenant de 5,20 m, inférieure à 5,44 m . C’est donc la courbe 2 qu’il faut utiliser . Le centre de gravité G est situé à 700 mm du mât . L’ordonnée correspondant à l’abscisse 700 mm sur la courbe 2 est 1 400 kg ; c’est la charge maximale que l’on peut gerber . Par conséquent, il est impossible de gerber la charge de 1 800 kg à une hauteur de 5,20 m .

Deux risques liés à l’utilisation des grues mobiles

•Répondreauxquestions

1. Une flèche de grue trop longue déplace le centre de gravité du système en dehors de la base de sustentation .

2. Le conducteur doit sortir les quatre poutres des stabilisateurs pour augmenter l’aire de la base de sustentation .

Tester ses connaissancesQ.C.M.1 . : B ; 2 . : C ; 3 : C ; 4 . : B ;5 . : B et C ; 6 . : A et B .

Tester ses capacités1. Centre de gravité d’un cylindreLe centre de gravité est situé au milieu du segment qui joint les centres des deux bases du cylindre .

2. Centre de gravité d’un anneaua. Le centre de gravité est situé au centre de l’an-neau .b. Le centre de gravité ne se trouve pas dans la matière de l’anneau .

3. Centre de gravité d’une plaque triangulaireIl faut tracer les médianes du triangle pour détermi-ner son centre de gravité .

4. Choisir la bonne réponseLe poids d’un corps se mesure avec un dynamomè-tre .La valeur du poids d’un corps s’exprime en newton .

5. Lire un appareil de mesureLe poids de l’objet suspendu en (a) est 4 N .Le poids de l’objet suspendu en (b) est 2,05 N .

6. Recopier et compléterLe poids d’un corps est représenté par un segment fléché

P dont l’origine correspond au centre de gra-

vité, la direction et le sens sont ceux du poids et la longueur est proportionnelle à la valeur du poids .

7. Représenter le poids d’une boule de pétanque.Le segment fléché a pour origine le point G, pour direction la verticale de G et de sens vers le bas .

Avec l’échelle choisie le segment fléché a pour longueur :

0,70,2

= 3,5 cm .

8. Stabilité d’un équilibreLe guéridon est moins stable que la table basse, car son centre de gra-vité est plus haut et sa base de sus-tentation plus réduite .

9. Basculement frontalLa charge à déplacer a été mal positionnée : la verti-cale du centre de gravité du système tombe en avant de la base de sustentation formée par la zone inté-rieure aux quatre roues . Le chariot bascule autour des roues avant .

G

P

Les exercices

Page 30: COLLECTION DURANDEAU SCIENCES

CHAPITRE 7 - Comment éviter le basculement d’un corps ? • 29

Appliquer le cours10. Mesure du poidsa. L’appareil de mesure photographié est un dyna-momètre .b. Il faut multiplier la lecture par le facteur 0,1 pour obtenir le poids . c. L’objet suspendu pèse : 3,5 × 0,1 = 0,35 N .

11. Calcul du poids à partir de la massea. Dans la relation P = m.g, P représente la valeur du poids de l’objet, m, la valeur de la masse et g, celle de l’intensité de la pesanteur .b. P s’exprime en newton, m en kilogramme et g en newton par kilogramme .c. La valeur du poids de l’objet est

P = 0,75 × 9,8 = 7,35 N .

12. Représentation du poids d’une chargea. Le bloc de béton a une forme rectangulaire . Il pos-sède deux axes de symétrie . Lorsqu’un solide homo-gène présente un axe de symétrie, son centre de gravité se trouve sur cet axe . Le centre de gravité se trouve donc à l’intersection des deux axes de symétrie . Il coïn-cide aussi avec les deux diagonales du rectangle .b. P = m . g = 500 × 10 = 5 000 N .c.

Point

d’application

Droite

d’action

Sens Valeur

P G Verticale

De haut

en bas5 000 N

G

P

A

13. Poids d’un panneau solairea. Le poids du panneau est :P = m .g = 40 × 10 = 400 N .b. Voir ci-contre (échelle : 50%) .

14. Montée sur une échelle simple inclinéea. Lorsque la personne monte sur l’échelle, le centre de gravité du système échelle-personne s’élève .

b. Si l’échelle n’est pas suffisamment inclinée, la base de sustentation est réduite . Lors de travaux de perçage, la personne exerce une action contre le mur . En réaction, elle a tendance à s’éloigner du mur et la verticale du centre de gravité peut facilement sortir de la base de sustentation ce qui entraîne le bascule-ment de l’échelle .

15. Équilibre d’un objet lestéDans la position 1 :a. la base de sustentation est le point O ;b. la verticale du point G rencontre le point O ;c. l’objet reste en équilibre .Dans la position 2 :d. la base de sustentation est le segment AB ;e. la verticale du centre de gravité ne rencontre pas la base de sustentation ;f. l’objet ne peut pas rester en équilibre, il se redresse .

Utiliser ses connaissances16. Inclinaison de l’échellea. Voir ci-contre (échelle : 50%).

Graphiquement on trouve :

BHM = 71° .

b. ABH BHM = .

tan,

, .ABHAHAB

= = =72 5

2 8

ABH = 70,3° .

17. Stabilisateursa. Le chef de chantier reproche au grutier de ne pas avoir sorti les deux stabilisateurs de la grue .b. Cette négligence peut avoir pour conséquence un renversement de la grue .c. Sur la photo de la page 83, on observe que le grutier n’a pas sorti les stabilisateurs . La base de sustentation était trop réduite et lorsque la grue a été déployée, la verticale du centre de gravité est sortie de la base de sustentation et la grue s’est renversée .

18. Le tracteur et l’épareusea. Lors du déploiement de l’épareuse, le centre de gravité du système tracteur-épareuse se déplace vers l’épareuse .b. Pour que le système ne se renverse pas, la verticale du centre de gravité doit rester entre les quatre roues du tracteur .c. Le fait de gonfler les roues du tracteur à l’eau ou de placer des masses à l’avant permet de diminuer le déplacement du centre de gravité lorsqu’on déploie l’épareuse .

G

P

A

H M

B

Page 31: COLLECTION DURANDEAU SCIENCES

30 • CHAPITRE 8 - Comment soulever un objet ?

19. Plaque de chargea. Pour une hauteur de 6,3 m c’est la courbe (2) qui donne le résultat . Pour une distance de 600 mm, la charge maximale est de 1 000 kg, soit 1 tonne .b. Pour une hauteur de 4,9 m, c’est la courbe (1) qui donne le résultat . Non le chariot ne peut pas élever une charge de 1,2 tonne dont le centre de gravité est situé à 0,7 m de la fourche, car le maximum est de 1 000 kg, soit une tonne .

20. Les grues mobilesa. Au niveau de la grue, le centre de gravité doit res-ter entre les stabilisateurs .b. La charge maximale que la grue peut soulever pour une portée de 18 m est 1,7 tonne .c. Si la grue doit soulever une masse de 20 tonnes, on doit choisir une longueur de flèche de 8,6 m .

8 comment soulever un objet ? HS1

Les activités

Activité 1 Comment caractériser et représenter une action mécanique ?

•MatérielDifférents documents présentant des situations où s’exercent des actions mécaniquesUn aimantUne bille d’acierUn dynamomètre circulaire

Expérience1Présenter différents documents montrant des situations où s’exercent des actions mécaniques de contact et à distance . Amener les élèves à préciser, qui exerce l’action et qui la subit, ainsi que les effets qui en découlent .On peut placer une bille d’acier sur une goulotte et la faire rouler près d’un aimant pour montrer qu’il existe des actions mécaniques à distance .

Expérience2Analyser l’action mécanique exercée par la main sur le fil du dynamomètre, afin de préciser les quatre caractéristiques d’une force .

•Réponsesauxquestions

Expérience1

1. Dans le doc . 1, ce sont les câbles qui exercent l’action et le container qui la subit . Les câbles sont au contact du container . L’effet provoqué est le déplacement du container .

Dans le doc . 2, c’est l’aimant qui exerce l’action et la bille qui la subit . L’effet provoqué est une modification du mouvement . Cette action s’effectue à distance .

Dans le doc . 3, c’est la presse d’emboutissage qui exerce l’action et la tôle qui la subit . Cette action provoque la déformation de la tôle .

Expérience2

2. Le point d’application de la force est le point P ; sa droite d’action, la direction du fil ; son sens, vers la droite et sa valeur 4 newtons .

3. Il faut déterminer les quatre caractéristiques d’une action mécanique : le point d’application, la droite d’action, le sens et la valeur pour la représenter par une force .

Page 32: COLLECTION DURANDEAU SCIENCES

CHAPITRE 8 - Comment soulever un objet ? • 31

Activité 2 Quelles sont les conditions d’équilibre d’un objet soumis à deux ou trois forces ?

•MatérielDeux dynamomètres magnétiques avec leur fil muni d’un crochetUne plaque de polystyrèneUne plaque triangulaire dont le centre de gravité est matérialiséUn support magnétique avec un fil à plomb

Expérience1Rechercher les caractéristiques des deux forces s’exerçant sur la plaque, en particulier, leur direction, leur sens et leur valeur .

Expérience2Maintenir la plaque triangulaire en équilibre à l’aide de deux dynamomètres . Positionner le fil à plomb afin qu’il passe par le centre de gravité de la plaque . Il matérialise la direction du poids de la plaque . Vérifier que les directions des deux forces et du poids sont concourantes .

•Réponseauxquestions

Expérience1

1. Les forces exercées en A et B ont même droite d’action, même valeur mais des sens opposés .

Expérience2

2. Les droites d’action des trois forces P ,

F1 et

F2 sont concourantes .

Remarque sur l’essentiel du cours

L’étude des vecteurs n’étant pas au programme de mathématiques de la classe de seconde professionnelle, nous avons supprimé toutes les expressions vectorielles concernant :- le principe des actions mutuelles ;- l’équilibre d’un solide soumis à deux ou à trois forces .

Tester ses connaissancesQ.C.M.1 : A ; 2 : B ; 3 : C ; 4 : C ; 5 : B ; 6 : C .

Tester ses capacités1. Le ressorta. L’action mécanique exercée par la main sur le res-sort est une action de contact .b. Elle est ponctuelle, car elle s’exerce à l’extrémité du ressort .

2. Actions mécaniques

Action mécanique Type d’action Effets

1 . Action de l’aimant sur la bille en acier Action à distance, répartie sur toute la

bille

Modification de la trajectoire de la bille

2 . Action des câbles sur le container Action de contact localisée Mise en mouvement du container

3 . Action de la presse d’emboutissage

sur la tôle .

Action de contact répartie Déformation de la tôle

4 . Action des haubans sur le tablier du

pont .

Action de contact localisée . Maintien du tablier au repos

3. Le dynamomètreLes caractéristiques de la force sont :- le point d’application : le point P ;- la droite d’action : l’horizontale ;- le sens : vers la droite ;- la valeur : 6 N .

NEW

TON

S2 4 6 8 10 P

Fmain/tige

(échelle : 50%)

Les exercices

Page 33: COLLECTION DURANDEAU SCIENCES

32 • CHAPITRE 8 - Comment soulever un objet ?

4. Représenter une force

C Fcorde/crochet

5. Le marteau

Fmarteau/clou

O

6. Prévoir un état d’équilibrea. Pour qu’un solide soumis à deux forces soit en équilibre, il faut que ces deux forces aient :

- même droite d’action ;

- même valeur ;

- des sens opposés .

b. Le seul solide en équilibre est le solide 2 qui pré-sente toutes les propriétés requises . Dans le schéma 1 les forces n’ont pas même droite d’action ; dans le schéma 3, les forces ont même droite d’action, des sens opposés mais n’ont pas la même valeur ; dans le schéma 4, les deux forces ont même sens .

7. Un état d’équilibre.a. Un solide soumis à deux forces est en équilibre si ces deux forces ont :

- même droite d’action ;

- même valeur ;

- des sens opposés .

b. Le solide (S) est en équilibre, car les deux forces ont des droites d’action identiques, même valeur : 5 N et des sens opposés .

8. Dynamique des forcesC’est le système (b) qui est en équilibre, car le dyna-mique des forces est fermé .

Appliquer le cours9. Réaliser un inventaire des forces

F1 : force exercée par Alain sur la corde (C) notée

F1 =

FA/C .

F2 : force exercée par Bernard sur le sol (S) notée

F2 =

FB/S .

F3 : force exercée par Alain sur le sol (S) notée

F3 =

FA/S .

F4 : force exercée par Bernard sur la corde (C) notée

F4 =

FB/C .

10. Réaction du supporta. Les deux forces qui s’exercent sur la boule sont le poids de la boule et la réaction du plan .b. Tableau des caractéristiques des forces :

Point

d’application

Droite

d’actionSens Valeur

Poids de

la bouleG Verticale

De haut

en bas3 N

Réaction

du planO Verticale

De bas

en haut3 N

c.

P

G

O

R

11. Tension du ressorta. Les deux forces qui s’exercent sur la boule sont le poids

P et la tension

T du ressort .

b. Tableau des caractéristiques des forces :

Point

d’application

Droite

d’action

Sens Valeur

P G Verticale

De haut en

bas5 N

T A VerticaleDe bas en

haut5 N

T

G

P

A

12. Boules électrisées en interactiona. Le principe des actions mutuelles dit que les deux

forces FA/B et

FB/ A ont même droite d’action, même

valeur mais des sens opposés :

(échelle : 50 %)

Page 34: COLLECTION DURANDEAU SCIENCES

CHAPITRE 8 - Comment soulever un objet ? • 33

b.

(A) (B)FB/A FA/B

c. Les sens de FA/B et de

FB/ A sont opposés .

Les valeurs sont égales à 2 N .

13. Force magnétique

a. La force

TC /B exercée par le câble sur la bille est appelée la tension du câble .b. Tableau des caractéristiques des forces :

Point

d’application

Droite

d’actionSens Valeur

P G Verticale

De haut

en bas4 N

FA/B G Horizontale

Vers la

gauche2,3 N

TC /B P 4,6 N

c. Dynamique des forces :

d. Graphiquement, la

valeur de la force FA/B

égale 2,3 N et celle de

TC /B

égale 4,6 N .

e. Cos 30° =

PT

=

32

d’où

T = 2P

33

= 4,62 N

F = ½ T = 2,31 N .

Utiliser ses connaissances14. Livraison d’un colisa. La valeur du poids de la gazinière est :P = m . g = 80 × 10 = 800 N .b. Tableau des caractéristiques des forces :

Point

d’application

Droite

d’actionSens Valeur

P G Verticale

De haut en

bas800 N

F1 A 420 N

F2 B 420 N

c. Dynamique des forces :d. Graphiquement la longueur des

vecteurs F1 et

F2 mesure 2,1 cm .

Leur valeur est donc :200 × 2,1 = 420 N .On peut alors reporter ces valeurs dans le tableau des caractéristiques des forces .e. La législation n’est pas respectée, car les livreurs doivent exercer une force de 420 N, supérieure à celle pré-vue par la législation (350 N) .

15. Équilibre d’une échellea. b.Voir ci-contre (échelle : 50 %) :

c. Puisque l’échelle est en équi-libre, les droites d’action des

trois forces P ,

FM /E et

FS/E sont

concourantes . La droite d’action de

FS/E est donc AC .

d. Tableau des caractéristiques des forces :

Point

d’application

Droite

d’actionSens Valeur

P G Verticale

De haut

en bas1 000 N

FM /E B Horizontale

De droite

à gauche120 N

FS/E A 1 000 N

e. Dynamique des forces :

Graphiquement on trouve :

FM /E mesure 0,6 cm,

soit 0,6 × 200 = 120 N ;

FS/E mesure 5 cm soit

5 × 200 = 1 000 N .

On peut alors reporter ces valeurs dans le tableau des caractéristiques des forces .

FA/B

TC/B

P

30°

P

F2

F1

P

20°

B

G

A

Cx

P

FS/E

FM/E

Page 35: COLLECTION DURANDEAU SCIENCES

34 • CHAPITRE 8 - Comment soulever un objet ?

16. Résistance d’un câblea. Tableau des caractéristiques des forces :

Point

d’application

Droite

d’actionSens Valeur

P G Verticale

De haut

en bas4 600 N

F1 A 4 000 N

F2 A 2 300 N

b. Dynamique des forces(échelle : 50%) :

c. Graphiquement la longueur de

P mesure 9,2 cm .

La valeur maximale du poids que l’on peut suspendre est donc :9,2 × 500 = 4 600 N .On peut retrouver cette valeur par le calcul dans le demi-triangle équilatéral PF1F2 :

P = 2 F1

33

= 2 × 4 000 x

33

= 4 618 N .

17. Tension des élinguesa. Tableau des caractéristiques des forces pour un angle a = 60° :

Point

d’application

Droite

d’actionSens Valeur

P G Verticale

De haut

en bas1 000 N

F1 E 580 N

F2 S 580 N

Dynamique des forces (échelle : 50%) :

P

H

Horizontale

F1/60° F1/45° F1/30°

Graphiquement sur le dynamique des forces on trouve

F1/60° = 5,8 × 100 = 580 N .On peut retrouver ces valeurs par le calcul :

F1/60° = 2 × 500 ×

33

= 577 N .

b. Valeurs des forces de F1 et

F2 pour différentes

valeurs de a :

a 30° 45° 60°

Valeur de P 1 000 N 1 000 N 1 000 N

Valeur de F1 1 000 N 700 N 580 N

Valeur de F2 1 000 N 700 N 580 N

c. Les valeurs des forces sont les plus importantes pour un angle de 30° . Ce résultat est général : les efforts à supporter par les élingues sont d’autant plus importants que l’angle entre l’élingue et la pièce à soulever est plus petit .

18. Le gymnaste aux anneauxa. Le poids du gymnaste est :

P = m . g = 78 × 10 = 780 N .b. Tableau des caractéristiques des forces s’exerçant sur le gymnaste :

Point

d’application

Droite

d’actionSens Valeur

P G Verticale

De haut

en bas780 N

T1 A 400 N

T2 B 400 N

c. Dynamique des forces :

Graphiquement on trouve T1 = T2 = 4 cm

Valeur de

T1 : 4 × 100 = 400 N .

Valeur de

T2 : 4 × 100 = 400 N .

F2

F1

P

T2

75°

T1

P

Page 36: COLLECTION DURANDEAU SCIENCES

CHAPITRE 9 - Comment soulever facilement un objet ? • 35

9 comment soulever facilement un objet ? HS1

Les activités

Activité 1 Quelle est l’influence du bras de levier d’une force ?

•MatérielUn point de fixationUn ressortUn axe de rotationUne barre à trousUn repèreUn dynamomètre

•Réponsesauxquestions

1. Lorsque l’équilibre de la barre est réalisé, le produit F .d est constant . Son unité est le newton-mètre .

2. Lorsque le bras de levier augmente, la valeur de la force diminue .

3. En exerçant la même force, Julie soulève le meuble dans le cas C, car cette situation présente le plus grand bras de levier de la force

F . Plus le bras de levier est grand, plus on peut soulever facilement un

meuble .

Activité 2 Comment calculer le moment d’un couple de forces ?

•MatérielpourunposteUn point de fixationUn ressortUn axe de rotationUne barre à trousUn repèreDeux dynamomètres

•Réponsesauxquestions

1. Lorsque les couples de forces ont le même effet sur l’équilibre, leurs moments sont égaux .

2. La position de l’axe de rotation n’intervient pas dans le calcul du moment d’un couple de forces .

3. Lorsque les bras de levier d des couples de forces augmentent, les valeurs F des forces diminuent . Pour des moments de couples égaux, les grandeurs F et d sont inversement proportionnelles .

Page 37: COLLECTION DURANDEAU SCIENCES

36 • CHAPITRE 9 - Comment soulever facilement un objet ?

Tester ses connaissancesQ.C.M. 1 : A et C ; 2 : C ; 3 : A, B et C ; 4 : A ; 5 : A .

Tester ses capacités1. À moment constanta. Les forces ont le même effet sur la barre parce que leurs moments sont égaux .b. Lorsque le bras de levier diminue, la force aug-mente .

2. Moment d’une force

a. Lorsqu’on utilise la relation F O

/ = F .d, F O

/ (N .m) est le moment de F

par rapport au point O,

F (N) représente la valeur de la force et d (m) est la distance de la droite d’action de la force à l’axe (bras de levier) .b. Le tableau suivant :

SituationsBarrière

de parkingTreuil Cric

Valeur de la force P = 500 N P = 100 N F = 20 N

Sens de rotation

∙ ou ⤺ ⤺ ∙ ∙Mesure du bras de

levier

OG

= 0,30 m

OT

= 0,1 m

OA

= 35 cm

Moment de la force par

rapport à l’axe150 N .m 10 N .m 7 N .m

3. Quel est le couple ?a. On a un couple de forces dans le cas c .b. Les autres schémas ne représentent pas un couple de forces :- en a, les forces n’ont pas la même valeur ;- en b, les forces ont le même sens ;- en d, les forces n’ont pas des droites d’action parallèles .

4. Moment d’une couple de forcesa. Lorsqu’on utilise la relation ℳC = F.d,- ℳC (N .m) est le moment d’un couple de forces

( F1

,F2

),

- F (N) la valeur commune des deux forces,- d (m) la distance séparant les droites d’action des deux forces .b. Le tableau suivant :

Situations Tournevis Tire-

bouchon

Cardan

Distance AB 6 mm 8 cm0,06 m ou

6 cm

Valeur de la

force F 20 N 5 N 40 N

Situations Tournevis Tire-

bouchon

Cardan

Sens de rotation

∙ ou ⤺ ∙ ⤺ ∙Moment du

couple ℳc0,120 N .m 0,40 N .m 2,4 N .m

5. Arrache-clou

a. F O

/ = F .OB = 15 × 0,80 = 12 N .m .

R O

/ = 0 N .m (la droite d’action de la force passe

par l’axe) .

F O’/ = F ’ .OA = 120 × 0,10 = 12 N .m .

b. On a F O

/ = F O’/ . Cette égalité traduit l’équi-Cette égalité traduit l’équi-

libre du pied de biche .

6. Mauvaise postureInventaire des moments des forces par rapport à O :

P O

/ = P .OB = 300 × 0,6 = 180 N .m ; Sens ∙

f O1

/

= f1 .OC = 250 × 0,15 = 37,5 N .m ; Sens ⤺ R O

/ = 0 N .m

F O1

/ = F1 .OA = 2 850 × 5 × 10-2 = 142,50 N .m ;

Sens ⤺À l’équilibre, on a P O

/ =

f O1

/

+ F O1

/ .

Appliquer le cours7. Couple de serragea. Le couple de serrage correspond au moment maximal de la force exercée par la main sur le man-che de la clé dynamométrique .

b. Valeur de la force maximale :

F = F O

d

/ =

5,20,18

= 28,9 N .

8. Système de freinagea. Moment de F

par rapport à O :

F O

/ = F .d = 40 × 0,27 = 10,8 N .m .

b. À l’équilibre, on a F O

/ = F O''/ : F "

est la force

exercée par le liquide de frein sur le piston .

La valeur de F ''

est :

FdF O"

'/=

=

10,80,03

= 360 N .

Par interaction, on déduit que la valeur de la force exercée par le piston sur le liquide de frein est égale-ment de 360 N : F ’ = 360 N .

c. On a

F 'F

=

36040

= 9 donc la valeur de la force F

a

été multipliée par 9 (cela correspond au rapport

dd '

des bras de levier) .

Les exercices

Page 38: COLLECTION DURANDEAU SCIENCES

CHAPITRE 9 - Comment soulever facilement un objet ? • 37

9. Pinces de forgea. L’inventaire des moments par rapport à l’axe sur une demi-pince est :

F O1

/ = F1

× 0,60 = 30 N .m ; F O’ /2

= F ’2 × 0,05 ;

R O

/ = 0 .

On a F O1

/ =

F O’ /2

soit F ’2 =

300,05

= 600 N .

La valeur de la force exercée par une mâchoire est de 600 N .

b. ( F1

,F2

) ne constituent pas un couple de forces, car

ces deux forces n’ont pas des droites d’action distinc-tes .

Utiliser ses connaissances10. Centre de gravité

a. Moment de F1

par rapport à O :

F O1

/ = 320 × 1,80 = 576 N .m ; sens ⤺ .

En appliquant la condition d’équilibre on a :

F O1

/ = P O

/ . (Le moment de

F2

par rapport à O

est nul, car la droite d’action de F2

passe par O) .

Le moment de P par rapport à O est égal à : 576 N .m ;

Sens ∙ .b. Soit d la distance séparant le point O de la verticale passant par G .Le moment de

P par rapport à O est :

P O

/ = P .d = 800 .d .

On a 800 .d = 576 soit d =

567800

= 0,71 m .

En position debout, la distance du sol au centre de gravité du sportif est de : 1,80 – 0,71= 1,09 m .

11. Levage avec palana. Nombre de brins n = 2 ; P = 5 N ; F = 2,5 N .

On retient la relation F =

Pn

, car 2,5 =

52

.

b. On a : n = 6 brins .c. Poids de la charge : P = m.g = 60 × 10 = 600 N .

La valeur de la force F est : F =

P6

=

6006

= 100 N .

12. Manutention sans risquea. À l’équilibre, l’inventaire des moments des forces par rapport à O donne :

P O

/ = P .OB = 300 × 0,35 = 105 N .m (sens ∙) .

f O1

/

= f1 .OC = 250 × 0,15 = 37,5 N .m (sens ⤺) .

F O2

/ = F2 .OA = 105 – 37,5 = 67,5 N .m (sens ⤺) .

R O

/ = 0 .

b. La valeur de la force F2

exercée sur le bas de la

colonne est :

F2 = F O2

0 05

/

, =

67,50,05

= 1 350 N .

Le rapport des deux valeurs des forces est :

28501350

= 2,1 .

Dans cette étude, la force exercée sur le bas de colonne est 2,1 fois plus faible lorsqu’on a les jambes pliées que lorsqu’elles sont tendues .

13. Porte-personnea. Distance à l’axe : pour P

, on a OB = 1,40 m et pour

F

on a OA = 0,40 m .b. Valeur de la force F

:

P O

/ = F O

/ donne 600 × 1,40 = F × 0,40

soit F =

8400,40

= 2 100 N .

14. Conducteur d’engin de chantier (situation-problème)Plaçons le point B à l’intersection de la droite d’action

du poids P1

et du sol et calculons les moments des

forces exercées sur le tracteur par rapport à A .

- le moment de P1

est : P A1

/ = m1 .g .AB

= 2 300 × 10 × 1,6 = 36 800 N .m (sens ⤺) ;

- le moment de P2

est : P A2

/ = m2 .g .AC

= 2 500 × 10 × 1,5 = 37 500 N .m (sens ∙) ;- en A la force de contact exercée par le sol sur la roue a un moment nul .

P A1

/ < P A2

/ : le chargeur de rochers bascule

autour du point A .

15. Dans la scierie (situation-problème)Ce problème ouvert a de nombreuses solutions ; on écarte celle avec la planche, car plus le bras de levier est grand, plus on soulève facilement la scie (d’autres arguments sur la flexibilité et la fragilité de la planche peuvent être avancés) .a. Première solution : en position d’équilibre ins-table (les verticales passant par les centres de gra-vité de la scie et d’Arthur passent par les extrémités du madrier) . Il s’agit de déterminer la distance OB à laquelle on place le point d’appui O . On pose OB = d .

BM

3,15

OC

AG

PA

Pm

R

Ps

6,30

d

Page 39: COLLECTION DURANDEAU SCIENCES

38 • CHAPITRE 10 - Quels sont les composants d’une eau minérale ?

On note : Pm

le poids du madrier ;

Ps

le poids de la

scie ; PA

le poids d’Arthur (g = 10 N/kg)

On a : P OA

/ = PA .g .d = 300d (sens ∙)

P Om

/ = m.g .(d – 3,15) = 400(d – 3,15) (sens ∙)

P Os

/ = ms . g .(6,30 – d) = 2 300 (6,30 – d) (sens ⤺)

R O

/ = 0 .

À l’équilibre P Os

/ = P Om

/ + P OA

/ donne

2 300(6,30 – d) = 300d + 400(d – 3,15)3 000 d = 15 750 soit d = 5,25 m .OB = 5,25 m et OC = 1,05 m .

b. Autre proposition : on peut améliorer la stabilité en avançant le madrier vers la gauche de 15 cm et en déplaçant Arthur de 10 cm par exemple : les vertica-les, en G et A, ne passent plus par les extrémités du madrier .

BM

3,15

O’C

AG

PA

Pm

R

Ps

6,30

0,150,10

d’

La même démarche que précédemment permettrait de calculer la position du point d’appui . Il va sans dire que l’exercice est difficile ; nous recommandons de le résoudre lors d’une activité dirigée avec l’aide du professeur . Cela pourra donner l’occasion de faire réviser la résolution des équations du premier degré à une inconnue .

10 Quels sont les composants d’une eau minérale ? HS2

Les activités

Activité 1 Comment identifier quelques ions présents dans une eau minérale ?

•MatérielQuatre tubes à essai sur un porte-tubesUne coupelleUne pipette simpleUn bouchon avec un tube à dégagement adapté

•ProduitsUne solution de nitrate d’argentUne solution de chlorure de baryumUne solution d’oxalate d’ammoniumUne eau minérale (eau Hépar par exemple)Du sulfate de cuivre anhydreUne bouteille d’eau minérale gazeuse (eau Perrier par exemple)De l’eau de chaux

•Réponseauxquestions

1. Les ions identifiés dans l’eau minérale sont :- les ions chlorure de formule Cl– (précipité blanc avec le nitrate d’argent) ;- les ions sulfate de formule SO4

2– (précipité blanc avec le chlorure de baryum) ;- les ions calcium de formule Ca2+ (précipité blanc avec l’oxalate d’ammonium) .

2. L’eau minérale contient de l’eau, car le sulfate de cuivre anhydre est devenu bleu .

Page 40: COLLECTION DURANDEAU SCIENCES

CHAPITRE 10 - Quels sont les composants d’une eau minérale ? • 39

3. Le dégagement gazeux trouble l’eau de chaux . L’eau minérale gazeuse contient du dioxyde de carbone dissous .

4. Une eau minérale contient :- de l’eau mise en évidence avec le sulfate de cuivre anhydre ; - des ions en solution caractérisés avec des réactifs ; - du dioxyde de carbone lorsqu’elle est gazeuse, mis en évidence avec l’eau de chaux .

Activité 2 Comment modéliser des molécules ?

•MatérielUne boîte de modèles moléculaires

•Réponseauxquestions

1. Dans une molécule, les atomes sont matérialisés par des boules colorées : une boule blanche représente un atome d’hydrogène ; une boule noire un atome de carbone ; une boule rouge un atome d’oxygène ; une boule jaune un atome de soufre ; une boule verte un atome de chlore ; une boule bleue un atome d’azote .

2. Les liaisons covalentes entre deux atomes sont matérialisées par des tiges, une tige par liaison .

3. La molécule de dihydrogène (a) est composée de 2 atomes d’hydrogène unis par une liaison covalente . Sa formule brute est H2 .La molécule de chlorure d’hydrogène (b) est composée de 1 atome d’hydrogène et de 1 atome de chlore, unis par une liaison de covalence . Sa formule brute est HCl .La molécule de méthane (c) est composée de 1 atome de carbone et de 4 atomes d’hydrogène ; chaque atome d’hydrogène est uni à l’atome de carbone par une liaison de covalence . Sa formule brute est CH4 .La molécule de dioxygène (d) est composée de 2 atomes d’oxygène unis par deux liaisons de covalence . Sa formule brute est O2 .

4. Un atome d’hydrogène réalise une liaison de covalence .Un atome de chlore réalise une liaison de covalence .Un atome de carbone réalise quatre liaisons de covalence .Un atome d’oxygène réalise deux liaisons de covalence .

Les documents

Comment établir le modèle de Lewis d’une molécule ?

•Réponseauxquestions

1. Il manque un électron à un atome d’hydrogène pour obéir à la règle du duet .Il manque deux électrons à un atome d’oxygène pour obéir à la règle de l’octet .

2. Dans le modèle de Lewis de la molécule d’eau :- le tiret reliant l’atome d’oxygène à un atome d’hydrogène représente une liaison de covalence simple ;- chaque atome d’hydrogène porte un doublet d’électrons et l’atome d’oxygène porte quatre doublets d’électrons .

3. Dans le modèle de Lewis de la molécule de dioxygène :- les deux tirets reliant les deux atomes d’oxygène représentent une double liaison de covalence ;- chaque atome d’oxygène porte quatre doublets d’électrons .

Page 41: COLLECTION DURANDEAU SCIENCES

40 • CHAPITRE 10 - Quels sont les composants d’une eau minérale ?

Tester ses connaissancesQ.C.M.1 : A et C ; 2 : A et C ; 3 : B ; 4 . B ; 5 : C ;

6 : B ; 7 : B .

Tester ses capacités1. Test des ions chlorurea. On utilise le nitrate d’argent pour identifier les ions chlorure dans une eau minérale .

b. On obtient le précipité blanc n°3 .

2. Tests des ions calcium et des ions sulfatea. Pour identifier les ions calcium, on utilise l’oxalate d’ammonium ; pour identifier les ions sulfate, on uti-lise le chlorure de baryum .

b. Lors de ces deux tests, on obtient un précipité blanc .

3. Test de reconnaissance de l’eauPour mettre en évidence la présence d’eau dans une boisson, on verse quelques gouttes de boisson sur du sulfate de cuivre anhydre qui devient bleu en pré-sence d’eau .

4. Test de reconnaissance du dioxyde de carbonePour mettre en évidence le dioxyde de carbone dis-sous dans l’eau de Badoit :

- on place un bouchon muni d’un tube à dégage-ment sur le goulot d’une bouteille d’eau de Badoit ;

- on plonge le tube à dégagement dans de l’eau de chaux contenue dans un tube à essai ou dans un verre à pied ;

- on agite la bouteille afin que le dégagement gazeux soit plus important ;

- on observe le trouble de l’eau de chaux .

5. L’atome d’oxygène

Le modèle de Lewis de l’atome d’oxygène est —••

|O6. La molécule d’eauLe modèle de Lewis de la molécule d’eau est :

H

O

H .

7. Le saccharoseLe saccharose est composé de 12 atomes de carbone, de 22 atomes d’hydrogène et de 11 atomes d’oxy-gène .

8. L’ion hydrogénocarbonatea. L’ion hydrogénocarbonate est composé d’un atome d’hydrogène, d’un atome de carbone et de trois atomes d’oxygène .b. Cet ion porte une charge négative .

9. Formule des ions

Ion sodium • • Cu2+

Ion potassium • • Cl–

Ion cuivre • • K+

Ion sulfate • • Mg2+

Ion chlorure • • SO42−

Ion magnésium • • Na+

10. Ion calciumUn atome de calcium qui a perdu deux électrons devient un ion calcium de formule Ca2+ .

Appliquer le cours11. Symbole des atomesLes symboles incorrects sont : magnésium : Mg ; sodium : Na ; potassium : K ; azote : N .

12. Structure de l’atomea. Le symbole chimique du potassium est K, celui du sodium est Na .b. Le noyau d’un atome de potassium comporte 19 protons et 39 – 19 = 20 neutrons ; son nuage électro-nique contient 19 électrons .Le noyau d’un atome de sodium comporte 11 pro-tons et 23 – 11 = 12 neutrons ; son nuage électronique contient 11 électrons .

13. Atome et ion magnésiuma. L’atome de magnésium possède 2 + 8 + 2 = 12 élec-trons au total .b. Sa couche externe contient 2 électrons .c. L’ion magnésium a pour formule Mg2+, car il pro-vient d’un atome de magnésium ayant perdu les deux électrons de sa couche externe M pour obéir à la règle de l’octet .

14. Ion oxydea. La structure électronique de l’atome d’oxygène est K2L6 .b. La couche externe L de l’atome d’oxygène contient 6 électrons .c. L’ion oxyde a pour formule O2–, car il provient d’un atome d’oxygène ayant gagné deux électrons afin de compléter sa couche externe L à huit électrons pour satisfaire la règle de l’octet .

Les exercices

Page 42: COLLECTION DURANDEAU SCIENCES

CHAPITRE 10 - Quels sont les composants d’une eau minérale ? • 41

15. L’ion carbonateLa formule de l’ion carbonate est CO3

2– .

16. Structure électronique

Atome de sodium 1123Na • • K2L8

Ion sodium Na+ • • K2L1

Atome de chlore 1735Cl • • K2L8M8

Ion chlorure Cl– • • K2

Atome de lithium 37Li • • K2L8M7

Ion lithium Li+ • • K2L8M1

17. Rechercher la formule d’un ion monoatomique

AtomeOxygène

816 O

Magnésium

1224 Mg

Fluor

919 F

Aluminium

1327 Al

Nombre

d’électrons8 12 9 13

Structure

électroniqueK2L6 K2L8M2 K2L7 K2L8M3

Nombre

d’électrons

sur la couche

externe

6 2 7 3

Formule

de l’ion

correspondant

O2– Mg2+ F– Al3+

18. Caractérisation des ions chlorure a. Pour caractériser les ions chlorure, Cyril a utilisé une solution de nitrate d’argent .

b. Il observe un précipité blanc dans les tubes à essai contenant de l’eau de Vittel, de l’eau Volvic et de l’eau du robinet .

c. On trouve donc des ions chlorure dans les eaux minérales de Vittel et Volvic ainsi que dans l’eau du robinet .

d. On n’obtient pas de précipité avec l’eau déminé-ralisée, car elle ne contient pas d’ions en solution .

19. Composition d’une eau minéralea. Formules des ions composant l’eau minérale : sodium : Na+ ; potassium : K+ ; calcium : Ca2+ ; magné-sium : Mg2+ ; bicarbonate : HCO3

– ; chlorure : Cl– ; sul-fate : SO4

2–; fluorure : F– .

b. On identifie les ions calcium avec l’oxalate d’am-monium, les ions chlorure avec le nitrate d’argent et les ions sulfate avec le chlorure de baryum . Dans les trois tests, on observe un précipité blanc .

c. Sur l’étiquette est mentionné que l’eau minérale Saint-Yorre est une eau minérale naturelle gazeuse . Le gaz dissous est du dioxyde de carbone pouvant être identifié avec de l’eau de chaux qui se trouble en sa présence .

20. Modèles moléculaires compactsEau : 5 ; dioxygène : 3 ; dioxyde de carbone : 2 ; dihy-drogène : 4 ; chlorure d’hydrogène : 1 .

21. Modèles de Lewis de quelques molécules

Molécules Eau DichloreChlorure

d’hydrogène

Modèles

de LewisH

O

HCICI H CI

Molécules Dioxygène Dioxyde de carbone

Modèles de

Lewis O O O C O

Molécules Méthane Ammoniac

Modèles de

Lewis H H

H

H

CH H

H

N

Utiliser ses connaissances22. En plomberiea.

Nom de l’élément Cuivre Étain

Symbole chimique Cu Sn

Nombre de protons 29 50

Nombre d’électrons 29 50

Nombre de neutrons 34 70

b. Le pictogramme signifie que le décapant utilisé par les plombiers est un produit corrosif .

23. Chromagea. L’atome de chrome comporte autant d’électrons que de protons, donc 24 électrons dans son nuage électronique .b. L’ion chrome, de formule Cr3+, provient d’un atome de chrome qui a perdu 3 électrons ; l’ion chrome comporte donc 24 protons dans son noyau et 21 électrons dans son nuage électronique .

24. Cellules photovoltaïquesa. La structure électronique du silicium est : K2L8M4 .b. La couche externe M du silicium comporte 4 élec-trons .c. Le modèle de Lewis de l’atome de silicium est •

• ••

Si .

d. L’atome de silicium est tétravalent, car il possède quatre électrons célibataires sur sa couche externe, il est susceptible de créer quatre liaisons de covalence .

Page 43: COLLECTION DURANDEAU SCIENCES

42 • CHAPITRE 11 - Comment déterminer la concentration d’une substance dans une solution ?

25. Paludiera. Structure électronique de l’atome de sodium : K2L8M1 ; celle de l’atome de chlore : K2L8M7 .

b. L’atome de sodium perd un électron (1 électron célibataire sur la couche M) pour saturer sa couche externe à huit électrons (règle de l’octet) et donne l’ion sodium de formule Na+ .

c. L’atome de chlore gagne un électron (7 électrons sur la couche M) pour saturer sa couche externe à huit électrons (règle de l’octet) et donne l’ion chlo-rure de formule Cl– .

26. JardinierLes ions mis en évidence dans la bouillie bordelaise sont :

- les ions cuivre II de formule Cu2+, car on a obtenu un précipité bleu avec la soude ;

- les ions sulfate de formule SO42–, car lors du test avec

la solution de chlorure de baryum, on a obtenu un précipité blanc .

27. Liquide d’usage couranta. On effectue le test de reconnaissance de l’eau avec le sulfate de cuivre anhydre .

b. En contact avec de l’eau, le sulfate de cuivre anhy-dre (blanc) devient bleu .

c. Les liquides contenant de l’eau sont : la limonade, le lait, le vinaigre blanc .

Le white spirit ne contient pas d’eau .

28. Pneus gonflés au diazotea. La représentation respectant la règle de l’octet est

N N , car chaque atome d’azote comporte quatre doublets d’électrons .

b. Le modèle de Lewis du diazote est donc

N N .

c. Les deux atomes d’azote sont unis par une triple liaison de covalence matérialisée par la mise en com-mun de trois doublets d’électrons .

29. Matières plastiquesa. La molécule d’éthylène comporte 2 atomes de car-bone et 4 atomes d’hydrogène .

b. La formule brute de l’éthylène est C2H4 .

c. Le modèle de Lewis de l’éthylène est CCH

H

H

H .

d. Chaque atome d’hydrogène est uni à un atome de carbone par une liaison de covalence simple .

Les atomes de carbone sont unis par une double liaison de covalence .

11 comment déterminer la concentration d’une substance dans une solution ? HS2

Les activités

Activité 1 Comment préparer une boisson sucrée par dissolution ?

•MatérielUne balance électronique à 0,1 gUne capsuleUne spatuleUn entonnoirUne fiole jaugée de 100 mLUne pipette simple

•ProduitsDu saccharose en poudre - Une pissette d’eau distillée

•Réponseauxquestions

1. On a dissous 1,7 g de saccharose dans 100 mL d’eau distillée .

2. La masse de saccharose que contiendrait un litre de solution est de 17 g .

3. La concentration est la masse d’une substance dissoute par litre de solution . Dans notre exemple, on dit que la concentration massique de la solution est de 17 g/L .

Page 44: COLLECTION DURANDEAU SCIENCES

CHAPITRE 11 - Comment déterminer la concentration d’une substance dans une solution ? • 43

Tester ses connaissancesQ.C.M. 1 . B et C ; 2 . C ; 3 . B ; 4 . B ; 5 . C ; 6 . C ; 7 . A et C ; 8 . B ; 9 . B .

Tester ses capacités1. SaccharoseM(C12H22O11) = 12 × M(C) + 22 × M(H) + 11 × M(O) = 12 × 12 + 22 × 1 + 11 × 16 = 342 g/mol .La masse molaire moléculaire du saccharose est de 342 g/mol .

Activité 2 Comment déterminer le caractère acide, basique ou neutre d’une solution ?

•MatérielTrois capsulesTrois petits béchersTrois tubes à essaiUn agitateur de verreDu papier pHUn pH-mètre, une pissette d’eau, du papier filtre fin (papier Joseph)

•ProduitsEau minérale VolvicVinaigre blancLiquide pour lave-vaisselleSolution de bleu de bromothymol

•Réponseauxquestions

1. 2. Classement des solutions par ordre de pH croissant : vinaigre blanc (pH = 2,1) ; eau de Volvic (pH = 7) ; liquide pour lave-vaisselle (pH = 13) .

3. Avec le bleu de bromothymol :- une solution acide devient jaune ;- une solution basique devient bleue ;- une solution neutre devient verte .

Les documents

Les dangers des liquides d’usage courant

•Réponseauxquestions

1. Avant d’utiliser le Destop, il faut lire attentivement l’étiquette portée sur le flacon afin de s’informer sur les dangers et les précautions à prendre lors de son utilisation .

2. a. Le pictogramme indique que le Destop est un produit corrosif . b. L’étiquette du produit Destop mentionne que le produit est corrosif, elle informe l’utilisateur qu’il faut éviter le contact du produit avec la peau et les yeux, car il provoque de graves brûlures . Les dangers indiqués sur l’étiquette sont donc en accord avec ceux indiqués par le pictogramme .

3. Le Destop contient de l’hydroxyde de sodium, produit très corrosif et dangereux .

4. Pour utiliser ce produit, il faut porter un vêtement de protection approprié, des gants et un appareil de protection des yeux et du visage .

5. Le Destop dissout intégralement les matières organiques, les cheveux, les graisses .

Les exercices

Page 45: COLLECTION DURANDEAU SCIENCES

44 • CHAPITRE 11 - Comment déterminer la concentration d’une substance dans une solution ?

2. Café sucréLa concentration massique de la solution de café est égale à 50 g/L .

3. Boisson sucréeLa concentration molaire de la boisson sucrée est de 0,1 mol/L .

4. Composition d’une eau minéraleL’étiquette indique qu’un litre d’eau d’Evian contient 80 mg d’ions calcium .Quantité de matière d’ions calcium contenus dans

80 mg : n =

mM

= 0,08040,1

≈ 0,002 mol .

La concentration molaire des ions calcium de l’eau d’Evian est donc égale à 0,002 mol/L .

5. Sirop de menthea. Cette opération est une dilution .b. Un litre de sirop de menthe contient 3 moles de saccharose dissous, donc 0,01 L de sirop contient :

0,03 mole de saccharose .La concentration molaire de la boisson préparée est

C =

nV

= 0,030,15

= 0,2 mol/L .

6. Dilution de l’eau de javela. On doit prélever 10 mL d’eau de javel pour réaliser 100 mL d’une solution diluée 10 fois .b. Pour effectuer cette dilution, il faut utiliser une pipette jaugée de 10 mL .

7. Acide ou basique ?La comparaison de la couleur (rouge orangée) prise par le papier pH au nuancier de la boîte, indique que le pH de la solution est de 2 .

8. Lire l’étiquette d’un produit ménagera. Le pictogramme indique que le produit ménager est nocif .b. L’étiquette mentionne que le CANARD WC ANTI-TARTRE contient de l’acide chlorhydrique .c. Il ne faut pas mélanger le produit avec de l’eau de javel, car du dichlore, un gaz dangereux, peut se libérer .d. Il faut éviter le contact avec la peau et les yeux, car le produit est irritant et provoque des lésions oculaires .

Appliquer le cours9. Grandeurs et unités

Grandeurs Unités

Masse • • mol/L

Volume • • g/mol

Quantité de matière • • g

Masse molaire • • mol

Concentration massique • • L

Concentration molaire • • g/L

10. Concentration molairea. La formule permettant de calculer la concentra-

tion molaire C d’une solution est : C =

nV

.

b. C : en mol/L ; n : en mol ; V en L .

11. Quantité de matièren = C . V = 0,5 × 0,1 = 0,05 mol .

La quantité de matière de l’espèce chimique dissoute est de 0,05 mol .

12. Dissolutionm = Cm . V = 400 × 0,5 = 200 g .

Il faut 200 g de sucre pour préparer le bocal de fruits au sirop .

13. Jus d’orangeQuantité de matière de sucre dans 18 g de sucre :

n =

mMsucre

=

18180

= 0,1 mol . C =

nV

= 0,10,1

= 1 mol/L .

La concentration molaire du sucre dans le jus de fruits est de 1 mol/L .

14. Solution de saccharosen = C . V = 0,25 × 0,5 = 0,125 mol .

D’où une masse m = n.M = 0,125 × 342 = 42,7 g .Il faut peser 42,7 g de saccharose pour préparer 500 ml d’une solution de saccharose de concentration :

C = 0,25 mol/L .

15. Lire l’étiquette d’une eau minéralea. Les valeurs chiffrées sur l’étiquette indiquent les concentrations massiques des ions en solution dansl’eau minérale St-Yorre .b. Sur l’étiquette, on lit : Sodium 1 708 mg/L . Dans 15 cL d’eau minérale, la masse de sodium est :

0,15 × 1 708 = 256,2 mg .On absorbe donc 256,2 mg de sodium lorsqu’on boit un verre de 15 cL d’eau minérale St-Yorre .

c. n(Na+) =

mM(Na)

= 1,70823

= 0,074 mol ;

n(Cl–) =

mM(Cl)

= 0,32235,5

= 9 ×10–3 mol .

Un litre d’eau minérale St-Yorre contient 0,074 mol d’ions sodium et 9×10–3 mol d’ions chlorure .d. Le sel est un composé ionique de chlorure de sodium . Cette eau riche en ions sodium et en ions chlorure a donc un goût salé .

16. Dureté d’une eau minéralea. L’étiquette indique les concentrations massiques en ions Ca2+ et en ions Mg2+ : C(Ca2+) = 203,8 mg/L et C(Mg2+) = 43,1 mg/L .

Dureté : D =

C(Ca2+ )

4+

C(Mg2+ )

2,4 =

203,84

+ 43,12,4

≈ 68,9 .La dureté de l’eau de Vittel est donc de 68,9 °HT .

Page 46: COLLECTION DURANDEAU SCIENCES

CHAPITRE 11 - Comment déterminer la concentration d’une substance dans une solution ? • 45

b. L’eau de Vittel est une eau dure, car sa dureté est supérieure à 15 °HT .c. L’utilisation abusive de l’eau de Vittel dans une cafetière électrique entraîne un entartrage (dépôt de calcaire) du circuit d’eau chaude .

17. Dilution d’un sirop de fraisea. Le volume de liquide dans le verre est V = 180 + 20 = 200 mL .b. Le sirop a été dilué 10 fois .c. 700 × 0,02 = 14 g . 20 mL de sirop contiennent donc 14 g de sucre ; le verre contient 14 g de sucre .

18. pH de quelques solutionsa. Coca-Cola (pH = 2,5) : acide ; Jus de citron (pH = 3,1) : acide ; eau de Vichy (pH = 8,2) : basique ; eau de Perrier (pH = 5,0) : acide ; lait (pH = 7,0) : neutre .b. La solution la plus acide est le Coca-Cola ; la plus basique est l’eau de Vichy .

19. Acidité des boissonsa. L’acidité de la boisson au Coca est due aux acidi-fiants : acide orthophosphorique et acide citrique .b. L’acidité de cette boisson peut être vérifiée en mesurant son pH avec un pH-mètre .

20. Ions et pHa. La solution 1 est la plus acide : elle contient donc davantage d’ions hydrogène H+ que d’ions hydroxyde HO– .b. La solution 3 est basique : elle contient davantage d’ions hydroxyde HO– que d’ions hydrogène H+ .c. La solution 2 est neutre : elle contient autant d’ions hydrogène H+ que d’ions hydroxyde HO– .

Utiliser ses connaissances21. Sportif

a. Cm =

mV

= 20,10,3

= 67 g/L .

La concentration massique des glucides dans la bois-son énergétique est donc de 67 g/L .

b. Cm =

mV

= 0,210,3

= 0,7 g/L .

La concentration massique du sodium dans la bois-son énergétique est donc de 0,7 g/L .

22. Comparer des eaux de boissona. L’eau de Badoit et l’eau de Volvic n’ont pas les mêmes propriétés, car leurs teneurs en sels minéraux sont différentes ainsi que leurs pH .b. Le résidu à sec à 180 °C indique la teneur globale (ou minéralisation totale) en sels minéraux en mg/L après chauffage à 180°C .

c. - L’eau de Badoit est la plus minéralisée, car son résidu à sec est le plus grand (1 200g/L) .- L’eau de Badoit est plus dure que l’eau de Volvic, car les concentrations en ions calcium et magnésium dans l’eau de Badoit sont plus élevées .- L’eau de Badoit (pH = 6) est plus acide que l’eau de Volvic (pH = 7), car son pH est plus faible .

23. Réaliser une dissolution au laboratoirea. Masse de pastilles d’hydroxyde de sodium à peser :

m = C .M .V = 0,5 × 40 × 0,1 = 2 g .Pour préparer la solution, il faut dissoudre 2 g de pas-tilles d’hydroxyde de sodium .b. Matériel de chimie nécessaire : une balance élec-tronique, une capsule, une spatule, un entonnoir, une fiole jaugée de 100 mL, une pissette d’eau distillée .

24. Hygiènea. Masse molaire moléculaire de l’hypochlorite de sodium :

M(NaClO) = 23 + 35,5 + 16 = 74,5 g/mol .b. Quantité de matière dans 4,25 g d’hypochlorite de sodium :

n =

mM

=

4,2574,5

≈ 0,06 mol .

Volume d’eau nécessaire : V =

nC

= 0,060,01

= 6 L .

Il faut 6 litres d’eau pour dissoudre une pastille de masse m égale à 4,25 g .

25. Lessive (développement durable)a. Les doses de lessive à choisir dépendent à la fois de la dureté de l’eau et de la saleté du linge à laver .Avec une eau dure, il faut davantage de lessive pour nettoyer le linge qu’avec une eau douce . b. Une eau dure contient beaucoup d’ions calcium et d’ions magnésium . Sa dureté est supérieure à 15 °HT .c. La dureté d’une eau dépend de la nature géolo-gique des terrains qu’elle a traversés . Ainsi, un sol crayeux ou calcaire donnera une eau dure, riche en ions calcium, alors qu’un sol granitique ou sablon-neux donnera plutôt une eau douce .d. La dureté de l’eau se constate principalement :- par la formation de dépôts insolubles avec les savons (« l’eau ne mousse pas ») dans les canalisations ;- par des taches grisâtres sur le linge ;- par l’entartrage (couche de calcaire) des tuyaux d’eau chaude .e. Les lessives contiennent des substances toxiques pour l’environnement . Les eaux de lessive sont dif-ficiles à dégrader dans les stations d’épuration . Ces stations n’existent pas partout, et en cas de rejet dans le sol, elles peuvent polluer la nappe phréatique .

Page 47: COLLECTION DURANDEAU SCIENCES

46 • CHAPITRE 12 - Que contient un soda ?

12 Que contient un soda ? HS2

Les activités

Activité 1 Comment identifier le glucose et les colorants contenus dans une boisson ?

Expérience1

•MatérielUn bécherDeux tubes à essaiUne plaque chauffante

•ProduitsUne solution de glucoseUne boisson au Coca-ColaUne solution de liqueur de Fehling

Expérience2

•MatérielUn bécher recouvert par une capsule ou une boîte de PétriUne feuille de papier filtre ou de papier à dessin (5 cm × 3 cm)Une règle, un crayon à papierTubes effilés ou piques en bois

•ProduitsUne solution d’eau saléeDu sirop de mentheUne solution aqueuse de tartrazine (E 102)Une solution aqueuse de bleu patenté (E 131)

•Réponseauxquestions

1. Expérience1 - Dans les deux tubes, la liqueur de Fehling a viré au rouge brique . Le Coca Cola contient donc du glucose .

2. Expérience2 a. On observe une migration de taches vers le haut de la feuille . Les taches jaunes migrent plus facilement et atteignent la même hauteur, les taches bleues migrent moins .b. Le sirop de menthe donne deux taches : une bleue et une jaune .c. On compare les positions des taches obtenues en mesurant les distances parcourues par chacune d’elles .

3. On identifie le glucose contenu dans un soda ou un sirop avec le test de la liqueur de Fehling : la liqueur de Fehling vire au rouge brique en présence de glucose .La chromatographie permet d’identifier les colorants .

Page 48: COLLECTION DURANDEAU SCIENCES

CHAPITRE 12 - Que contient un soda ? • 47

Activité 2 Comment extraire un arôme ?

Expérience1

•MatérielUn couteau à bout rondUn mixeurUn erlenmeyer avec un bouchonUne éprouvette graduée de 25 mLUn dispositif de filtration sous pression réduite (fiole à vide, trompe à eau, support, filtre Büchner, joint conique, papier filtre…)Une ampoule à décanterDeux béchers

•ProduitsUne orangeDu cyclohexaneDe l’eau distillée

Expérience2

•MatérielUn couteau à bout rondDeux supports avec noix de serrage, pincesUn élévateur à croisillonsUn chauffe-ballon thermostatéUn ballon de 250 mLUne tête de colonne, un condenseur à eau, une allonge coudéeUne éprouvette graduée de 50 mLPapier filtre

•ProduitsEau distilléeUne orange

•Réponseauxquestions

1. Expérience1a. Pour extraire les arômes des zestes d’orange, on broie et on mélange les zestes dans un solvant, ici le cyclohexane dans lequel les arômes à extraire sont très solubles .b. Le cyclohexane est un liquide inflammable ; il doit être manipulé à l’écart de toute flamme ou de toute source de chaleur .c. d(cyclohexane) < d(eau), la phase organique contenant le cyclohexane se trouve au-dessus de la phase aqueuse .d. Pour récupérer l’essence de limonène contenue dans le cyclohexane, il suffit de placer le bécher contenant la phase organique sous une hotte aspirante afin que le cyclohexane s’évapore, car il est très volatil .

2. Expérience2a. Les vapeurs d’eau chargées des arômes de zestes d’orange se condensent dans le tube central du réfrigérant, refroidi par un courant d’eau froide . Le distillat est recueilli à la sortie du réfrigérant .b. Le distillat présente deux phases : de l’eau et le limonène .c. Le limonène se trouve dans la phase supérieure . On utilise une ampoule à décanter pour séparer les deux phases et récupérer le limonène .

Page 49: COLLECTION DURANDEAU SCIENCES

48 • CHAPITRE 12 - Que contient un soda ?

Tester ses connaissancesQ.C.M.1 : B et C ; 2 : A et B ; 3 : A et C ; 4 : C ; 5 : A et C ; 6 . : B et C ; 7 : B et C .

Tester ses capacités1. Connaître le matériel de chromatographiea : cuve à chromatographie ; b : front de l’éluant ; c : phase fixe ; d : ligne de dépôt ; e : éluant ; f : couver-cle .

2. Analyser un chromatogrammeLe chromatogramme obtenu est le c, car le colorant bleu, plus soluble que le colorant rouge, migre plus haut . Les taches bleues migrent à la même hauteur, ainsi que les taches rouges .

3. Rapport frontala. h1 = 8 mm ; h2 = 18 mm ; h3 = 26 mm ; H = 31 mm .b. Les rapports frontaux sont :

R(C1) =

h1

H=

831

≈ 0,26 ; R(C2) =

h2

H=

1831

≈ 0,58 ;

R(C3) =

h3

H=

2631

≈ 0,84 .

Appliquer le cours4. Test du glucosea. On utilise la liqueur de Fehling pour mettre en évi-dence la présence de glucose dans une solution .b. Dans le tube 1, on a versé une solution sucrée contenant du glucose, car la liqueur de Fehling a viré au rouge brique .

5. Réaliser une chromatographiea. La phase fixe est la feuille de papier à chromato-graphie .b. L’éluant est le mélange de cyclohexane et d’éther . Lors de sa migration sur le support fixe, l’éluant entraîne les constituants de l’huile essentielle de peaux d’oranges .c. Le trait (a) matérialise le front de l’éluant, le trait (b) la ligne de dépôt .d. L’huile essentielle de peaux d’oranges contient du limonène et du citral, car les deux taches correspon-dant au dépôt n° 4 migrent à la même hauteur que les taches correspondant aux dépôts n° 1 de limonène et n° 3 de citral .

6. Rapport frontala. Rapports frontaux :

colorant E 102 : Rf =

hH

=

3548

≈ 0,73 ;

colorant E 131 : Rf =

hH

=

1848

≈ 0,38 .

b. Rapport frontal du colorant : Rf =

hH

=

2155

≈ 0,38 .

On trouve le rapport frontal du colorant E 131 .

Le sirop contient du bleu patenté . Sa couleur est bleue patenté .

7. Ampoule à décantera. Une ampoule à décanter est utilisée pour extraire une substance par solubilisation dans un solvant . Elle permet de recueillir la phase organique contenant le solvant et l’espèce à extraire .

b. 1er cas : le cyclohexane, moins dense, surnage .

cyclohexane

eau

2e cas : le dichlorométhane, plus dense, est situé dans la partie inférieure .

eau

dichlorométhane

8. Choix d’un solvant et sécuritéa. Le composé C, non miscible à l’eau, ne peut pas se mélanger avec l’eau, il forme un mélange hétérogène avec l’eau .

b. Le solvant ne doit pas être miscible avec l’eau : on écarte l’éthanol, on a le choix entre le cyclohexane et le dichlorométhane .

Le composé C doit être très soluble dans le solvant : le solvant ne peut donc pas être le cyclohexane . Le sol-vant choisi est donc le dichlorométhane .

c. Les pictogrammes apposés sur le flacon de dichlo-rométhane signifient que le solvant est irritant et inflammable .

Pour éviter une réaction au contact avec la peau, les yeux et les voies respiratoires, il faut porter des gants de protection, des lunettes et manipuler le dichloro-méthane sous une hotte aspirante .

Les exercices

Page 50: COLLECTION DURANDEAU SCIENCES

CHAPITRE 12 - Que contient un soda ? • 49

Comme ce solvant est inflammable, il faut le conser-ver et l’utiliser à l’écart de toute flamme ou de source de chaleur .d. Le dichlorométhane, plus dense que l’eau, se situe sous la phase aqueuse .

eau

dichlorométhane

9. Principe de l’hydrodistillationa. L’hydrodistillation est un procédé d’extraction et de séparation de certaines substances organiques par entraînement à la vapeur d’eau .b. 1 : Mélange de zestes d’orange et d’eau ; 2 : chauffe-ballon ; 3 : ballon ; 4 : thermomètre ; 5 : eau ; 6 : réfri-gérant ; 7 : éprouvette ; 8 : distillat ; 9 : eau ; 10 : huile essentielle .c. La vapeur d’eau produite au cours de l’ébullition entraîne les substances à extraire non solubles dans le liquide vaporisé (ici, l’eau) .d. Le réfrigérant condense les vapeurs et fournit un distillat, mélange d’eau de condensation et d’huile essentielle contenant le limonène à extraire .e. L’huile essentielle contenant le limonène est recueillie dans l’éprouvette à la sortie du réfrigérant .

Utiliser ses connaissances10. Au distributeur de boissonsa. Test de présence de l’eau : on verse quelques gout-tes de chaque boisson sur du sulfate de cuivre anhy-dre contenu dans une capsule ou une boîte de Pétri . Si le sulfate de cuivre anhydre devient bleu, la boisson contient de l’eau .Test du glucose : dans un tube à essai, on chauffe au bain-marie quelques millilitres de chaque bois-son avec 1 à 2 millilitres de liqueur de Fehling . Si le mélange prend une couleur rouge brique, la boisson contient du glucose .Test du dioxyde de carbone : faire barboter dans de l’eau de chaux le gaz dissous dans chaque boisson . Si l’eau de chaux devient trouble, la boisson contient du dioxyde de carbone .b. Le Coca Cola Light contient de l’eau et du dioxyde de carbone .

Le Coca Cola classique contient de l’eau, du glucose et du dioxyde de carbone .c. Le Coca Cola est qualifié de Light lorsqu’il ne contient pas de glucose .d. Boire régulièrement des boissons trop sucrées peut être néfaste pour la santé en raison du risque de diabète et d’obésité que ces boissons peuvent entraî-ner .

11. Chromatogramme de boissons a. Les rapports frontaux sont :

- acide lactique : Rf =

3843

≈ 0,88 ;

- acide citrique : Rf =

2243

≈ 0,51 ;

- acide malique : Rf =

2443

≈ 0,56 ;

- acide tartrique : Rf =

1543

≈ 0,35 .

b. Les acides citrique et malique ont des rapports frontaux voisins et sont difficilement différentiables .c. Le jus est le n° 3 : il contient de l’acide tartrique et de l’acide malique mais pas d’acide lactique .d. Les vins contiennent tous de l’acide lactique . Le vin n° 2 est celui dont la fermentation est la plus avan-cée, car tout l’acide malique a été transformé en acide lactique .

12. Chromatographie des glucidesa. La phase fixe est la plaque chromatographique, elle est composée d’une couche de silice hydratée sur une feuille d’aluminium .b. La phase mobile est l’éluant, mélange composé d’éthanol, d’acide éthanoïque et de butanone .c. On a révélé la plaque chromatographique afin de colorer les taches de sucre initialement invisibles . d. Les rapports frontaux sont :

- glucose G : Rf =

2054

≈ 0,37 ;

- saccharose S: Rf =

654

≈ 0,11 ;

- fructose F : Rf =

1654

≈ 0,30 .

e. Le jus d’orange est composé de glucose et de sac-charose .f. L’étiquette indique que le jus d’orange contient du saccharose et du sirop de glucose, ce qui est en accord avec les résultats obtenus avec la chromato-graphie du jus d’orange .

Page 51: COLLECTION DURANDEAU SCIENCES

50 • CHAPITRE 13 - Que se passe-t-il lorsque des produits d’usage courant réagissent entre eux ?

13 Que se passe-t-il lorsque des produits d’usage courant réagissent entre eux ? HS2

Les activités

Activité 1 Quelle réaction se produit-il entre l’acide chlorhydrique et le calcaire ?

•MatérielDeux tubes à essaiUn tube à dégagementUn verre à piedUn rouleau de papier indicateur de pHUne coupelle

•ProduitsAcide chlorhydrique diluéUne solution de nitrate d’argentDes morceaux de calcaireEau de chaux

Expérience1Elle permet d’identifier les ions contenus dans l’acide chlorhydrique afin de pouvoir écrire sa formule ionique . L’identification de l’ion chlorure Cl– a déjà été réalisée au chapitre 10, l’étude de l’acidité a été revue au chapitre 11 .

Expérience2Elle permet d’identifier le gaz qui se dégage lors de la réaction entre l’acide chlorhydrique et le calcaire . Cette réaction a été déjà vue au collège en cours de SVT pour reconnaître les roches calcaires .

•Réponsesauxquestions

Expérience1

1. Le pH de l’acide chlorhydrique est voisin de 2 . L’ion responsable de l’acidité est l’ion H+ .

2. L’ion chlorure est identifié par le test au nitrate d’argent .

3. La formule ionique de l’acide chlorhydrique est : (H+ + Cl–) .

Expérience2

4. Il se produit entre l’acide chlorhydrique et le tartre la même réaction qu’entre l’acide chlorhydrique et le calcaire . L’acide chlorhydrique attaque le tartre avec dégagement de dioxyde de carbone qui trouble l’eau de chaux .

Activité 2 Quelles sont les caractéristiques d’une réaction chimique ?

•MatérielUne bouteille d’eau minérale de un litre avec son bouchonUne balance électronique

•ProduitsAcide chlorhydrique diluéDes morceaux de calcaire

Expérience1Elle permet de vérifier que les atomes se conservent au cours d’une réaction chimique .

Page 52: COLLECTION DURANDEAU SCIENCES

CHAPITRE 13 - Que se passe-t-il lorsque des produits d’usage courant réagissent entre eux ? • 51

Expérience2La bouteille avec son bouchon est pesée avec l’acide chlorhydrique et le calcaire . Cette réaction permet de vérifier la conservation de la masse .

•Répondreauxquestions

Expérience1

1. Les atomes contenus dans les réactifs sont les atomes d’hydrogène, de chlore, de calcium, de carbone et d’oxygène .

2. Les atomes contenus dans les produits sont les mêmes mais ils sont disposés différemment .Les atomes se conservent au cours de la réaction chimique .

Expérience2

3. La bouteille gonfle, car il se dégage du dioxyde de carbone qui ne peut pas s’échapper .

4. La masse des réactifs et des produits se conserve au cours de la réaction chimique . La masse des réactifs diminue mais la masse des produits augmente si bien que la masse totale ne change pas .

documentation

Risques et sécurité

1. La concentration de l’acide étant inférieure à 25 %, les risques encourus par sa manipulation sont :- R36 irritant pour les yeux .- R37 irritant pour les voies respiratoires .- R 38 irritant pour la peau .

2. Les principaux moyens de sécurité à mettre en œuvre sont :- S24 Éviter le contact avec la peau ;- S25 Éviter le contact avec les yeux ;- S26 En cas de contact avec les yeux, laver immédiatement et abondamment avec de l’eau et consulter un spécialiste ;- S28 Après contact avec la peau, se laver immédiatement et abondamment avec de l’eau ;- S36/37/39 Porter un vêtement de protection approprié, des gants et un appareil de protection des yeux et du visage .

Les exercices

Tester ses connaissancesQ.C.M.1 . : B ; 2 . : A ; 3 . : C ; 4 . : C ; 5 . : A ;

6 . : B .

Tester ses capacités1. Dilution de l’acide chlorhydriquea. La concentration de l’acide dilué étant inférieure à 25 %, les risques encourus par sa manipulation sont :

- R36 irritant pour les yeux ;

- R37 irritant pour les voies respiratoires ;

- R 38 irritant pour la peau .

b. Les principaux moyens de sécurité à mettre en œuvre sont :

- S24 éviter le contact avec la peau ;

- S25 éviter le contact avec les yeux . ;- S26 en cas de contact avec les yeux, laver immé-diatement et abondamment avec de l’eau et consul-ter un spécialiste ;- S28 après contact avec la peau, se laver immédia-tement et abondamment avec de l’eau ;- S36/37/39 porter un vêtement de protection approprié, des gants et un appareil de protection des yeux et du visage .

2. Combustion du carboneL’équation chimique de la réaction est :

C + O2 → CO2

3. Combustion du dihydrogèneL’équation chimique de la réaction est :

2 H2 + O2 →2 H2O

Page 53: COLLECTION DURANDEAU SCIENCES

52 • CHAPITRE 13 - Que se passe-t-il lorsque des produits d’usage courant réagissent entre eux ?

4. L’acide chlorhydrique dissout le tartreL’équation chimique de la réaction est :

2 (H+ + Cl–) + CaCO3 →(Ca2+ + 2Cl–) + H2O + CO2

5. L’acide chlorhydrique décape le ferL’équation chimique de la réaction est :

2 (H+ + Cl–) + Fe →(Fe2+ + 2Cl–) + H2

Appliquer le cours6. Écrire des équations chimiques avec des coefficients convenablesa. 3 Fe + 2 O2 → Fe3O4

b. 2 Al + Fe2O3 → 2 Fe + Al2O3

c. CH4 + 2 O2 → CO2 + 2 H2O

7. Chercher l’erreur.a. L’élève ne porte pas de blouse, ni de gants .b. L’élève pipette le liquide avec la bouche, ce qui est formellement interdit . Il doit utiliser une poire à pipeter .c. Les bouchons ou les compte-gouttes ne doivent pas rester sur la paillasse . Il faut reboucher les flacons de produits chimiques après utilisation .d. L’élève doit manipuler l’acide concentré avec des gants .Il faut toujours verser l’acide concentré dans l’eau et non le contraire . En effet, le dégagement de chaleur qui se produit au contact de l’eau et de l’acide peut provoquer des projections d’acide .e. L’élève n’a pas gardé les lunettes sur les yeux pour manipuler l’acide .f. L’élève ne doit pas garder les gants pour allumer des allumettes . Il doit fermer la boîte avant de frotter l’allumette .

8. Précautions d’emploia. Code des risques encourus :Provoque de graves brûlures : R 34b. Code des moyens de sécurité à mettre en œuvre :- S1/2 : conserver sous clé et hors de portée des enfants ;- S 24 et S 25 : éviter le contact avec la peau et les yeux ;- S 26 : en cas de contact avec les yeux, laver immé-diatement et abondamment avec de l’eau et consul-ter un spécialiste ;- S28 : après contact avec la peau, se laver immédia-tement et abondamment avec de l’eau et du savon ;- S36/37/39 : porter un vêtement de protection approprié, des gants et un appareil de protection des yeux et du visage ;- S 45 : en cas d’accident ou de malaise, consulter immédiatement un médecin (si possible lui montrer l’étiquette) .

c. L’acide contenu dans ce produit est de l’acide chlorhydrique .

9. Écrire les équations bilan

a. N2 + 3 H2 → 2 NH3

b. 2 Al + Fe2O3 → 2 Fe + Al2O3

c. 2 Na + Cl2 → 2 NaCl

d. 2 C2H2 + 5 O2 → 4 CO2 + 2 H2O

e. 4 Al + 3 O2 → 2 Al2O3

f. 2 Cu + O2 → 2 CuO

g. Ca + 2 H2O → Ca(OH)2 + H2

h. Fe2O3 + 3 CO → 2 Fe + 3 CO2

10. Conserver les atomes et les charges

a. Zn + 2 (H+ + Cl–) → (Zn2+ + 2 Cl–) + H2

b. Cu2+ + Fe →Cu + Fe2+

c. 3 Cu2+ + 2 Al → 3 Cu + 2 Al3+

d. 2 Al + 6 (H+ + Cl–) → 2 (Al3+ + 3 Cl–) + 3 H2

e. ClO– + 2H+ + Cl– → Cl2 + H2O

11. Vérifier la conservation des atomes

a. C3H8 + 5 O2 → 3 CO2 + 4 H2O

b. On trouve trois atomes de carbone, huit atomes d’hydrogène et dix atomes d’oxygène dans les réactifs et les produits .

c. Il y a bien conservation des atomes au cours de la réaction .

12. Vérifier la conservation des atomesa. Les réactifs mis en présence sont le fer et le dioxy-gène . Le produit obtenu est l’oxyde de fer .

b. La loi mise en évidence par cette observation est la loi de Lavoisier : la somme des masses des réactifs qui réagissent est égale à la somme des masses des produits formés .

c. L’équation de la réaction est :

3 Fe + 2 O2 → Fe3 O4 .

13. Vérifier la conservation de la charge électrique.a. Dans les réactifs, les ions positifs sont les ions hydrogène : 2 H+ et les ions négatifs sont les ions chlo-rure : 2 Cl– .

b. La charge totale des ions présents dans les réactifs est nulle : (2+) + (2–) = 0 .

c. Dans les produits, les ions positifs sont les ions magnésium : Mg2+ et les ions négatifs sont les ions chlorure : 2 Cl– .

d. La charge totale des ions présents dans les pro-duits est nulle : (2+) + (2–) = 0 .

e. La conservation de la charge électrique est satis-faite .

Page 54: COLLECTION DURANDEAU SCIENCES

CHAPITRE 13 - Que se passe-t-il lorsque des produits d’usage courant réagissent entre eux ? • 53

Utiliser ses connaissances14. Équation chimique et bilan molairea. Les réactifs de la réaction chimique sont l’éthanol de formule C2H6O et le dioxygène de l’air de formule O2 . Les réactifs sont le dioxyde de carbone de formule CO2 et l’eau de formule H2O .

b. L’équation de la réaction est :

C2H6O + 3 O2 → 2 CO2 + 3 H2O

c.

Équation bilan

C2H6O + 3 O2 → 2 CO2 + 3 H2O

Bilan molaire

1 mol 3 mol 2 mol 3 mol

d. Bilan molaire de la réaction : une mole d’éthanol réagit avec trois moles de dioxygène pour for-mer deux moles de dioxyde de carbone et trois moles d’eau .

15. Combustion du fera. L’équation de la réaction s’écrit :

3 Fe + 2 O2 → Fe3O4

b. Bilan molaire de la réaction : trois moles de fer réagissent avec deux moles de dioxygène pour former une mole d’oxyde de fer .

c. Nombre de moles de fer qui ont réagi

Équation bilan

3 Fe + 2 O2 → Fe3O4

Bilan molaire

3 mol 2 mol 1 mol

Nombre de moles

n 0,2 mol

n = 0 2 3

1, ×

= 0,6 mol .

d. Masse de fer qui a réagi : 0,6 × 56 = 33,6 g .

Masse molaire de l’oxyde : M = 3 × 56 + 4 × 16 = 232 g/mol .

Masse d’oxyde formé : 0,2 × 232 = 46,4 g

16. Élaboration de l’aciera. Équation chimique de la réaction :

Fe2O3 + 3 CO → 2 Fe + 3 CO2

b. Bilan molaire de la réaction : une mole d’oxyde de fer réagit avec trois moles de monoxyde de car-bone pour former deux moles de fer et trois moles de dioxyde de carbone .

c. Masse molaire de Fe2O3 :

M = 2 × 56 + 3 × 16 = 160 g/mol .

d. Masse de minerai nécessaire pour obtenir 11,2 tonnes de fer :11,2 tonnes de fer représentent :

11,256

× 106 = 2 × 105 moles de fer .

Équation bilan

Fe2O3 + 3 CO → 2 Fe + 3 CO2

Bilan molaire

1 mol 3 mol 2 mol 3 mol

Nombre de moles

n2 × 105

mol

Le nombre n de moles d’oxyde de fer utilisé est 105 moles .Masse de minerai : 160 × 105 = 16 × 106 g = 16 tonnes .

17. Dilution de l’acide chlorhydrique

a. Volume d’acide à mélanger : 5 ×

30100

= 1,5 L .

Volume d’eau à mélanger : 5 ×

70100

= 3,5 L .

b. Il faut verser en premier l’eau dans le seau, car on doit toujours verser l’acide dans l’eau .c. Après traitement, il faut rincer le seau abondam-ment .

18. Un peu de mathématiquesa. Le pictogramme signifie que le produit est irritant .b. La masse d’un litre de solution est :

1,12 × 1 000 = 1 120 g .c. Masse d’acide chlorhydrique contenu dans un litre de solution :

23100

× 1 120 = 257,6 g .

19. Le chalumeau oxyacétyléniquea. La molécule d’acétylène de formule C2H2 est com-posée d’atomes de carbone et d’hydrogène .b. L’équation de la réaction est :

2 C2H2 + 5 O2 → 4 CO2 + 2 H2O

c. L’équation de la combustion du fer dans le dioxy-gène est :

3 Fe + 2 O2 → Fe3O4

d. Si on brûle 6 moles de fer, il apparaît 2 moles de Fe3O4 .e. La masse de fer qui a réagi est :

6 × 56 = 336 g .Masse molaire de Fe3O4 : M = 3 × 56 + 4 × 16

= 232 g/mol .La masse d’oxyde qui est apparue est :

2 × 232 = 464 g .

Page 55: COLLECTION DURANDEAU SCIENCES

54 • CHAPITRE 14 - Comment préserver son audition ?

14 comment préserver son audition ? HS3

Les activités

Activité 1 Comment évolue le niveau d’intensité acoustique lorsque le récepteur s’éloigne de la source sonore ?

•MatérielUn GBF - Un haut-parleur - Un interrupteur - Un sonomètre - Une règle

Distance d entre le haut-parleur et le sonomètre (cm) 5 10 20 40

Niveau sonore L en dB (arrondi à l’unité) 80 74 68 62

•Réponsesauxquestions

1. Voir figure ci-contre.

2. Le niveau d’intensité acoustique diminue lorsque la distance entre la source et le récepteur augmente . Cette diminution est de 6 décibels chaque fois que la distance double .

3. S’éloigner de la source sonore constitue une bonne protection contre les sons .

Activité 2 Comment protéger son oreille ?

•MatérielUn GBF - Un haut-parleur - Un interrupteur - Un sonomètreDes PICB (protections individuelles contre les bruits)

•Réponsesauxquestions

1. 2. 3 . Les mesures et les réponses aux questions dépendent des PICB choisis .

4. L’utilisation de dispositifs de protection individuelle contre le bruit constitue une protection efficace contre le bruit .

Les documents

Le diagramme de Fletcher et Munson

•Réponsesauxquestions

1. Non, ces sons ne sont pas obligatoirement perçus de la même façon . Un son de 40 décibels et 5 000 hertz est perçu beaucoup plus fort qu’un son de 40 décibels et 100 hertz .

2. Une variation de 10 décibels produit plus d’effets avec un son de 100 hertz qu’avec un son de 3 000 hertz . Pour un son de 100 hertz, à partir du seuil d’audition, une variation de 10 décibels permet de couper quatre courbes isosoniques alors que pour un son de 3 000 hertz, on ne coupe que deux courbes .

0

60

65

70

75

80

10 20 4050

d (cm)

L (dB)

Page 56: COLLECTION DURANDEAU SCIENCES

CHAPITRE 14 - Comment préserver son audition ? • 55

3. Pour un son de 4 000 hertz, la douleur apparaît pour un niveau d’intensité sonore d’environ 113 décibels .

4. L’oreille humaine est plus sensible aux sons dont la fréquence est comprise entre 2 000 et 5 000 hertz .

La prévention contre les traumatismes auditifs

•Réponsesauxquestions

1. Deux amis assistent à un concert . Un seul d’entre eux utilise des PICB .

2. Le PICB utilisé dans la BD est un bouchon d’oreille .

3. Juste après le concert, la personne qui ne s’est pas protégée a les oreilles qui sifflent .

4. Le médecin diagnostique des acouphènes . Ce sont des sifflements permanents .

5. Sur l’affiche, on peut lire « Un seul concert peut vous ruiner la vie . » donc une seule exposition à un niveau sonore élevé peut avoir de graves conséquences .

6. Si une impression de surdité ou des sifflements persistent le lendemain d’un concert, il est conseillé de consulter un médecin . Cette consultation doit avoir lieu au plus tard dans les 72 heures après le concert .

Tester ses connaissancesQ.C.M.1 . B ; 2 . B, C ; 3 . B ; 4 . B ; 5 . B ; 6 . A ; 7 . A ; 8 . B, C .

Tester ses capacités1. L en fonction de da. Le niveau d’intensité acoustique L diminue lors-que la distance d entre la source sonore et le récep-teur augmente .b.

Distance de mesure d (m) 2 4 16 32

Niveau d’intensité acoustique L (dB)

98 92 80 74

2. Vrai – Fauxa. Faux : Le niveau d’intensité acoustique se note L .b. Faux : Le niveau d’intensité acoustique se mesure avec un sonomètre .c. Faux : Le niveau d’intensité acoustique diminue lorsque la distance croît .d. Vrai : Le niveau d’intensité acoustique décroît lorsque la distance augmente .e. Faux : Le niveau d’intensité acoustique augmente lorsque la distance diminue .

3. Mousse ou cireL’affaiblissement moyen provoqué par les PICB en

mousse est plus important que pour ceux en cire : les PICB en mousse sont les plus performants .

4. Comparaison de deux casquesa. Pour protéger de sons de 250 Hz, le casque nº 917B est plus adapté .

b. Pour protéger de sons de 1 000 Hz, le casque nº 924A est plus adapté .

Appliquer le cours5. Connaître et respecter les limitesa. L’unité du niveau d’intensité sonore est le décibel . Attention ! l’unité d’intensité sonore est le W .m-2 .

b. À partir de 85 décibels il existe un risque auditif .

c. Pour un baladeur à pleine puissance, la durée maximale journalière d’écoute est de 20 minutes .

En discothèque, la durée maximale journalière d’écoute est de 4 minutes .

d. Le bricolage ou la chasse peuvent exposer l’oreille à un risque avéré pour l’audition .

e. Assister sans protection à un essai moteur ou à une explosion peut être douloureux .

f. Non, avoir une conversation n’est pas dangereux pour l’audition .

g. Dans le cas « L = 90 dB pendant 3 h », la norme n’est pas respectée .

Dans le cas « L = 105 dB pendant 4 minutes », la norme est respectée .

Les exercices

Page 57: COLLECTION DURANDEAU SCIENCES

56 • CHAPITRE 14 - Comment préserver son audition ?

6. Vrai ou fauxa. Incomplet : La fatigue auditive provoque une gêne temporaire ou des acouphènes qui sont des siffle-ments permanents .b. Faux : Les acouphènes sont des sifflements per-manents entendus après une exposition à un bruit excessif .c. Vrai : La guérison d’un traumatisme auditif est impossible .d. Faux : Un déficit auditif peut n’être que tempo-raire si l’exposition n’a pas été trop importante .

7. Reculer pour mieux se protégerHugo mesurera 97 décibels à 4 mètres, 91 décibels à 8 mètres et 85 décibels à 16 mètres .

8. Mettre de l’ordreMesure L4 au point A, mesure L2 au point B, mesure L3 au point C et mesure L1 au point D .

9. Choisir le dispositif adaptéa. Pour protéger d’un son de 250 Hz, le casque 2 est le plus adapté .b. Pour protéger d’un son de 4 000 Hz, le casque 2 est le plus adapté .c. Pour les sons de fréquence 700 Hz : atténuation avec le casque 1 : 30 dB, atténuation avec le casque 2 : 30 dB .Pour les sons de fréquence 2 500 Hz : atténuation avec le casque 1 : 40 dB, atténuation avec le casque 2 : 35 dB .d. La gamme de fréquences pour laquelle le disposi-tif 1 est le plus adapté est [700 Hz ; 3 300 Hz] .

Utiliser ses connaissances10. S’éloigner pour se protéger (TICE)a.

b. Le niveau d’intensité acoustique diminue lorsque la distance entre la source sonore et le récepteur aug-mente .c. Si les passants circulent à moins de quatre mètres du marteau, ils doivent être protégés . S’ils circulent à moins de huit mètres, il leur est conseillé de se proté-ger . Au-delà de huit mètres, il n’y a plus d’obligation

mais il n’est pas conseillé de rester longtemps à proxi-mité du chantier .

11. Où se placer ?a. Le niveau d’intensité maximale pouvant être mesuré lors du concert, à un mètre des haut-parleurs, sans que la norme soit dépassée, est de 140 décibels, car l’atténuation entre 1 m et 10 m est de 20 dB .b. L’atténuation sera alors de 32,5 décibels . Le son mesuré à 40 mètres aura un niveau de 97,5 décibels .

12. La loia. L’objectif de la loi est de « protéger les personnes contre un niveau excessif de décibels pour la protec-tion des risques d’altération du système auditif » .b. Le niveau d’intensité sonore moyen fixé par la loi est de 105 décibels .c. Le niveau d’intensité sonore à ne pas dépasser qui a été fixé par la loi est de 120 décibels .d. Si un contrôle est effectué, le sonomètre sera placé au même endroit que le public .

13. Opérateur sur machinea. L’opérateur peut utiliser un casque ou des bou-chons d’oreille .b. Les dispositifs de l’exercice 3 permettent des affai-blissements supérieurs à 17 décibels pour toutes les fréquences, donc ils sont suffisants .

14. Se protéger 100 % du tempsa. • Si l’ouvrier enlève ses protections une minute dans sa journée de travail, la protection effective est de 23 dB . 100 – 23 = 77 dB . Si le niveau d’intensité sonore est de 100 dB, alors il ne court pas de risques .• Si l’ouvrier protège ses oreilles 98 % du temps, la protection effective est de 17,5 dB . 100 – 17,5 = 82,5 dB . L’ouvrier est dans une situation où il lui est conseillé de se protéger pour éviter une fatigue auditive .• Si l’ouvrier protège ses oreilles 100 % du temps, il ne risque rien .b. Si l’ouvrier ne se protège que 50 % du temps, le niveau d’intensité acoustique ne doit pas dépasser 87 + 3 = 90 dB .

15. Étude d’un audiogrammea. Audiogramme tenant compte du déficit auditif (seuil d’audibilité corrigé)

020406080

100120140

16 32 64 125 250 550 1 000 2 000 4 000 8 000 16 000

(Hz)

(dB) LE CHAMP AUDITIF HUMAIN

Seuil d’audibilité

Seuil de la douleur

Musique orchestrale

Conversation à 1 m

Page 58: COLLECTION DURANDEAU SCIENCES

CHAPITRE 15 - Comment peut-on décrire le mouvement d’un véhicule ? • 57

b. Non, ce patient ne sera pas gêné pour suivre une conversation à un mètre .

c. Oui, ce patient est gêné pour suivre un concert de musique orchestrale : sa perception des sons aigus est altérée .

16. Situation problèmeLes facteurs de risques sont :

- la durée lors de l’écoute (voir graphique du cours, page 170) ;

- le niveau d’intensité sonore (voir graphique du cours, page 170) ;

- la hauteur des sons écoutés . Les sons dont la fré-quence est proche de 4 000 Hz sont plus dangereux pour l’audition que les sons plus graves .

15 comment peut-on décrire le mouvement d’un véhicule ? T1

Les activités

Activité 1 Comment un enregistrement permet-il de connaître la nature d’un mouvement ?

•MatérielUn scooter, une bicyclette, une voiturette tirée par un filUn caméscope, une webcam reliée à un ordinateurUn logiciel de mécanique de traitement de l’image : Avistep, Aviméca,…Une règle ou un repère de 1 mètre (caméscope) ou 10 centimètres (webcam) pour un mobile proche

ExpérienceLe professeur peut faire réaliser les films et les traiter en projection à l’aide du logiciel .Chaque élève peut également traiter les films mis en commun sur un poste informatique individuel .

•Réponseauxquestions

1. Le scooter roule à vitesse constante dans le film 2 . Le repère mesure 1 m de longueur . Les distances entre deux croix jaunes consécutives sont de 0,42 m .

La vitesse moyenne du scooter est : 0,42 ÷ 1

25= 10,5 m/s soit 37,8 km/h .

2. Le film 3 correspond à un mouvement accéléré : à des intervalles de temps égaux à

125

de seconde correspondent des distances parcourues de plus en plus grandes .Le film 1 correspond à un mouvement ralenti : les distances parcourues sont de plus en plus petites .

3. Une chronophotographie ou un film vidéo permettent de déterminer la nature du mouvement d’un point d’un mobile et de réaliser des mesures et des calculs : distances, durées, vitesses moyennes .

Activité 2 Le mouvement d’un point d’un objet dépend-il du référentiel choisi ?

•MatérielUne bicycletteUn support d’atelier (on peut aussi renverser la bicyclette et la poser sur la selle et le guidon)Deux pastilles autocollantes

Page 59: COLLECTION DURANDEAU SCIENCES

58 • CHAPITRE 15 - Comment peut-on décrire le mouvement d’un véhicule ?

•Réponseauxquestions

Situation1

1. Le point G (le guidon) se déplace et a pour trajectoire un segment de droite .

2. Le point V (la valve) se déplace et décrit une courbe appelée cycloïde .

Situation2

3. Le point G est immobile par rapport au sol et par rapport à la bicyclette .

4. Le point V se déplace par rapport au sol et par rapport à la bicyclette : sa trajectoire est un cercle centré sur le moyeu de la roue .

5. La trajectoire d’un point d’un objet en mouvement dépend du référentiel choisi . Ainsi, le mouvement de la valve d’une roue se déplaçant sur le sol est une cycloïde si le référentiel choisi est le sol et un cercle si le référentiel choisi est le cadre de la bicyclette .

Les documents

Analyse d’un film vidéo

•Réponseauxquestions

1. Sur un caméscope, deux images successives sont séparées par

125

s ou 0,04 s .

2. Le référentiel généralement choisi est tout objet immobile sur le sol de la terre (référentiel terrestre) .

3. Pour calculer la vitesse moyenne du mobile, il faut réaliser l’étalonnage d’un objet de dimension connue, puis placer des croix colorées sur les différentes positions prises par un point du mobile .On peut aussi faire calculer les vitesses moyennes par le logiciel après avoir choisi les axes du repère .

Distances de sécurité

•Réponseauxquestions

1. Facteurs influant sur la distance de freinage :- l’état du véhicule ;- l’état de la chaussée ;- la vitesse du véhicule .

2. Lorsque la vitesse est multipliée par 2, la distance de freinage est multipliée par 4 ; lorsque la vitesse est multipliée par 4, la distance de freinage est multipliée par 16 .Ces résultats montrent que la distance de freinage dF est proportionnelle au carré de la vitesse v du véhicule . On a dF = k .v2 .

3. Pour une même vitesse du véhicule, la distance de freinage double sur route mouillée, .

4. Vitesse de la voiture exprimée en m/s :

v = 130 ÷ 3,6 ≈ 36,1 m/s .Lors du temps de réaction, le conducteur parcourt en 1,2 seconde :

36,1 × 1,2 = 43 m .D’où une distance d’arrêt sur route sèche :

dA = dR + dF = 43 + 95 = 138 m .

Sur route mouillée : d’A = 43 + 190 = 233 m .

Page 60: COLLECTION DURANDEAU SCIENCES

CHAPITRE 15 - Comment peut-on décrire le mouvement d’un véhicule ? • 59

Tester ses connaissancesQ.C.M.1 : C ; 2 : B et C ; 3 : A et C ; 4 : B et C ; 5 : B .

Tester ses capacités1. AscenseurRéférentiels possibles : le palier et Hubert .

2. Scooter et bicyclettea. Référentiel choisi : la route .

b. Référentiel choisi : la route .

3. Escalier mécaniquea. Paul et Nora sont en mouvement par rapport au sol .

Jim est en mouvement par rapport au tapis roulant .

b. Jim est immobile par rapport au sol . Paul et Nora sont immobiles par rapport au tapis roulant .

4. Trajectoires de billesTrajectoires : rectiligne en 1 ; circulaire en 3 ; quelcon-que en 2 .

5. Lancer de marteaua. La main tient le câble : la trajectoire de la boule est circulaire de M1 à M8 .

b. La main a lâché le câble : la trajectoire de la boule est rectiligne de M9 à M12 .

6. Chronophotographiea. Mouvement uniforme en 1 .

b. Mouvement ralenti en 3 .

c. Mouvement accéléré en 2 .

Appliquer le cours7. Unités de temps

a. 1 h = 60 min ; 1 min =

160

h = 0,017 h .

b. 15 min = 1

4 h = 0,25 h ;

20min = 1

3 h ≈ 0,33 h ;

30 min = 1

2 h = 0,5 h ;

45 min = 3

4 h = 0,75 h .

8. Convertir rapidementa. 110 km/h = 30,6 m/s .

b. 10 m/s = 36 km/h .

9. Horaires SNCFa. Corail : durée du trajet : 1 h 12 min ; TER : durée du trajet : 1 h 49 min .

Le trajet le plus rapide est celui du Corail .

b. La distance entre Pau et Lannemezan par le rail est de 78 km .Vitesses moyennes :

Corail : vmoyenne = 78

72≈ 1,083 km/min ou 65 km/h .

TER : vmoyenne = 78

109≈ 0,716 km/min ou 43 km/h .

10. Reconnaître un mouvementMouvement uniforme en 2 ; accéléré en 1 et 3 ; unifor-mément accéléré en 1 ; uniformément ralenti en 4 .

11. Déplacement d’une voiturea. Tableau

Date (s) 0 1 2 3 4 5

Vitesse (km/h)

72 81 89 98 106 116

Vitesse (m/s)

20 22,5 24,7 27,2 29,4 32,2

b. Représentation graphique de la vitesse en fonc-tion de la durée :

date (s)

Vitesse (m/s)

0

4

20

32

1 2 3 4 5

c. La vitesse est une fonction affine du temps : le mouvement de la voiture est uniformément accéléré .

Utiliser ses connaissances12. Porte-outil automatiséa. Positions du porte-pipetteb.

y (cm)

x (cm)

0

10

20

3

4 513

8,12

9

10,11

6 7

1,2

10 20 30 40

Les exercices

Page 61: COLLECTION DURANDEAU SCIENCES

60 • CHAPITRE 15 - Comment peut-on décrire le mouvement d’un véhicule ?

c. Les phases de repos sont : 1 à 2 ; puis 10 à 11 . La phase de mouvement rectiligne uniforme concerne les positions : 4 à 8 .

Lors de cette phase, la vitesse du porte-pipette est de :

v = 40

4 × 0,5= 20 cm/s .

13. Étude d’un freinagea. Origine des temps et des espaces : la position du motocycliste au début du freinage .

b. Le freinage dure 6 secondes : la position atteinte est de 108 m .

c. Le calcul des vitesses moyennes se calcule ainsi :

En C3 : = (D2-B2)/(D1-B1) ; on sélectionne la zone : C3 : M3, puis on active la fonction : recopier à droite .

d. À l’instant t = 0, v = 37 m/s soit 133 km/h .

La moto est immobile pour t = 6 s .

Le mouvement de la moto, lors de cette phase, est uniformément ralenti, car la vitesse v est une fonc-tion affine du temps t .

14. Distances de freinagea. La distance de freinage dépend de :

- l’état de la route ;

- l’état de la voiture ;

- la vitesse de la voiture .

b. Lorsque la vitesse double, la distance de freinage est multipliée par 4 ; Lorsque la vitesse triple, la dis-tance de freinage est multipliée par 9 .

c. Sur route sèche :

CS = v 2

2g .DF

= 33,32

2 × 9,8 × 72≈ 0,786 .

Sur route mouillée, CS est deux fois plus faible :

CS = 0,393 .

15. Distance de sécuritéa. Recommandation à respecter : laisser deux traits entre deux véhicules .130 km/h = 36,1 m/s . Donc 72 m sont parcourus en 2 secondes .b. Entre deux bandes blanches, il y a :

38 + 14 + 38 = 90 m .c. Cette distance est supérieure à 72 m : la sécurité est respectée .

16. Fête forainea. L’âne et le lama décrivent des trajectoires circulai-res .b. Longueur d’un tour :pour l’âne : L = π .D = π × 3 = 9,4 m ; pour le lama : L = π . D = π × 6 = 18,8 m .c. Vitesses moyennes des deux animaux :

l’âne : v A = 9,4

18≈ 0,52 m/s ;

le lama : v A = 18,8

18≈ 1,04 m/s .

Le lama se déplace deux fois plus vite que l’âne .

17. Parachutismea. Représentation graphique :

Distance (m)

temps (s)

0

100

50

10

1 2 3 4 5

b. Sur des intervalles de temps égaux, les distances parcourues ne sont pas égales ; elles augmentent avec le temps . Le mouvement n’est pas uniforme, mais accéléré .c. Pour t = 5 s et x = 121 m, on a :

a = 2x

t 2= 2×121

52= 9,7 m/s2 .

Cette valeur est voisine de 9,8 m/s2 .

Page 62: COLLECTION DURANDEAU SCIENCES

CHAPITRE 16 - Comment passer de la vitesse des roues à celle de la voiture ? • 61

16 comment passer de la vitesse des roues à celle de la voiture ? T2

Les activités

Activité 1 Que mesure un tachymètre ?

•MatérielUne platineUn moteur entraînant un disque translucide marqué d’une bande noireUn générateur continu 6/12V et un potentiomètre électroniqueUn tachymètre par contact ou par réflexionUn montage de photocoupleur (fourche électronique)Des fils de connexion

•MontageduphotocoupleurUn photocoupleur appelé aussi fourche optoélectronique est une association d’un émetteur (DEL infrarouge) et d’un détecteur (phototransistor ou photodiode) .Pour réaliser l’expérience, on peut le brancher ainsi :

470 Ω

+ 4 à 6 V

E

M

YSD

4,7 kΩ

•Réponseauxquestions

Expérience1

1. Les pics correspondent aux passages de la bande noire devant le capteur du photocoupleur .Notre disque comportait 6 bandes noires régulièrement espacées .

2. Dans notre expérience, la période du disque : T = 120 ms soit 0,12 s .

3. La fréquence de rotation du disque : n = 8,3 tr/s .

Expérience 2

4. Le tachymètre mesure la fréquence de passage d’une bande noire : 3 066 tr/min . Donc une fréquence de rotation du disque :

3 066 ÷ 6 = 511 tr/min .

5. Pour passer des tours par minute aux tours par seconde, on divise par 60 :

511÷ 60 ≈ 8,5 tr/s .

6. On retrouve à peu près la fréquence précédente : 8,3 tr/s .

Page 63: COLLECTION DURANDEAU SCIENCES

62 • CHAPITRE 16 - Comment passer de la vitesse des roues à celle de la voiture ?

Activité 2 Quelle est la relation entre la fréquence de rotation et la vitesse linéaire ?

•MatérielUne roue de bicyclette sur un support - Un compteur de bicyclette (cyclomètre)Un tachymètre - Un mètre pliant

•RésultatsPour la roue : R = 33,5 cm d’où une circonférence L = 210 cm .

v (km/h) v (m/s) N (tr/min) n (tr/s) 2π.R .n

1re allure 6,7 1,86 56 0,93 1,95

2e allure 11 3,05 90 1,50 3,15

3e allure 18 5,0 148 2,47 5,19

•Réponseauxquestions

1. L’expression 2 π .R correspond à la circonférence de la roue .

2. Pour chacune des trois allures, on a approximativement (les deux colonnes grisées) l’égalité : v = 2 π .R .n .

3. La relation entre la fréquence de rotation n d’une roue et la vitesse v de la bicyclette est : v = 2 π .R .n .

Les documents

La transmission du mouvement circulaire

•Réponsesauxquestions

1. Une bicyclette utilise la transmission par chaîne ; une voiture, certaines motos utilisent la transmission par pignons .

2. De n1 . D1 = n2 . D2, on sort : n2 =

n1 .D1

D2

= 1500 × 250100

= 3750 tr/min .

3. Le rapport de transmission est : RT =

nsortie

nentrée

= 37501500

= 2,5 .

Lorsque la poulie motrice effectue 1 tour, la poulie réceptrice effectue 2,5 tours .

La transmission de la moto Kawasaki ER-6

•Réponsesauxquestions

1. Pour un régime moteur n1 = 6 600 tr/min (en 4e) :- pour la transmission : moteur / arbre primaire :

n1 . Z1 = n2 . Z2 d’où :

n2 =

n1 .Z1

Z2

= 6600 × 4288

= 3150 tr/min

- pour la transmission : arbre secondaire / arbre primaire :

n3 . Z3 = n4 . Z4 d’où :

n4 =

n3 .Z3

Z4

= 3150 × 2730

= 2835 tr/min

- pour la transmission : roue arrière / arbre secondaire :

n5 . Z5 = n6 . Z6 d’où :

n6 =

n5 .Z5

Z6

= 2835×1546

= 924 tr/min

2. Le rapport de transmission total est : RT =

nsortie

nentrée

= 9246600

= 0,1400 . On retrouve la valeur de la fiche technique du fabricant .

Page 64: COLLECTION DURANDEAU SCIENCES

CHAPITRE 16 - Comment passer de la vitesse des roues à celle de la voiture ? • 63

Tester ses connaissancesQ.C.M.1 : A et B ; 2 : A et B ; 3 : A et C ; 4 : B ; 5 : B et C ; 6 : A et B .

Tester ses capacités1. Lecture d’une fréquence de rotationa. L’appareil est un tachymètre .b. Il est gradué en tour par minute .c. La fréquence de rotation est de : 35 × 100 = 3 500 tr/min .

2. Expériencea. Entre deux pics : 40 ms .b. Un tour de disque dure 40 ms ou 0,04 s .

D’où la fréquence de rotation : n = 1

T= 1

0,04= 25 tr/s .

3. Fréquencemètre de véloa. Le compteur affiche des tours par minute : RPM signifie « Rotation Per Minute » en anglais .b. n = 84 tr/min .

4. Mesures de fréquence de rotation et de vitesse linéairea. Vitesse du cycliste : 41,7 km/h . Fréquence de rota-tion des roues : 321 tr/min .b. Vitesse : 11,58 m/s ; fréquence : 5,35 tr/s .c. π .D .n = π × 0,69 × 5,35 = 11,60 m/s . On retrouve pratiquement la vitesse du cycliste affichée par le compteur .

5. Termes d’une expressiona. v : vitesse linéaire de l’outil, R : rayon de l’outil ; n : fréquence de rotation .b. Unités : v en mètre par seconde, R en mètre et n en tour par seconde .

6. Tondeuse électriquea. N = 2 790 ÷ 60 = 46,5 tr/s .b. V = π .D .n = π × 0,38 × 46,5 = 55,5 m/s .

7. Scie circulaire portative

a. Pour v = 30 m/s, nvD

= =×

=π π . ,

,300 19

50 3 tr/s .

b. Soit environ 3 000 tr/min .

8. Période et fréquenceLors du mouvement de rotation d’un objet autour d’un axe :- les trajectoires de chaque point de l’objet sont des cercles ; - l’objet effectue un mouvement uniforme si sa fré-quence de rotation est constante ;

- la durée pour effectuer un tour s’appelle la période ; - la période T d’une rotation est liée à la fréquence

n par la relation n = 1

T ; dans cette relation, T s’ex-

prime en seconde et n en tour par seconde .

9. Calculs de période et de fréquence

Période

T (s)1 2 0,01 4 ×10-4 0,2 40

Fréquence

de rotation

n (tr/s)

1 0,5 100 2 500 5 0,025

10. Vitesse de rotation a. n = 50 tr/s .b. n = 3 000 tr/min .

11. Cyclomètre a. L’écriture correcte de l’unité de vitesse : km/h .b. 18 km/h = 5 m/s .

c. nv

R= =

×=

25

2 0 352 27

π π . ,, tr/s . ou 136 tr/min .

12. Grandeurs et unités

Grandeur

physique

Symbole de la

grandeurUnité légale

Symbole de

l’unité

Période T seconde s

Fréquence de

rotationn

tour par

secondetr/s

Vitesse linéaire vmètre par

secondem/s

13. Calculs

Objet en

rotationv R n

Lame de scie

circulaire50,3 m/s 12 cm 4 000 tr/min

Pale

d’éolienne9,4 m/s 15 m 6 tr/min

Fraise de

Dremel13 m/s

3,9 ×10-3 m

ou 3,9 mm32 000 tr/min

Roue de moto 120 km/h 0,38 m14tr/s ou 840

tr/min

Satellite SPOT

II7 550 m/s 4 030 km

2,98 ×10-4 tr/s

ou ≈ 1 tr/h

14. Pour lire l’heurea. Les périodes : aiguille des heures : 12 h pour 1 tour ; aiguille des minutes : 1 h pour 1 tour ; aiguille des secondes : 1 min pour 1 tour .b. Les fréquences : respectivement :

1720

tr/min = 1,39 × 10-3 tr/min,

160

tr/min = 0, 017 tr/min et 1 tr/min .

Les exercices

Page 65: COLLECTION DURANDEAU SCIENCES

15. Satellites terrestres artificiels

a. Météosat : fréquence de rotation :

186 400

tr/s ;

vitesse linéaire : v = 2π .R .n = 2π × 4,2 × 107 × 1

86400

= 3 054 m/s ou 11 000 km/h .

b. La Station Spatiale Internationale (ISS) :

v = 27 700 km/h = 7 700 m/s .

Circonférence de l’orbite : L = 2π .R = 4,25 × 107 m

soit 42 500 km .

Fréquence de rotation :

n = v

2π .R= 7700

4,25×107= 1,81×10−4 tr/s

soit une période de 5 524 s ou 92 min .

16. Perçage dans l’acier

a. 270 tr/min = 4,5 tr/s .

b. Vitesse de coupe du foret : v = π . D . n

= π × 28 ×10-3 × 4,5 = 0,396 m/s soit 23,8 m/min .

c. Nombre de tours de foret pour avancer de 18 mm : 18 ÷ 0,23 = 78,2 arrondis à 79 tours .

Donc une durée de : 79 ÷ 4,5 = 17,6 s .

17. Le choix de la vitesse

a. Circonférence d’une roue : π .D = π × 0,69 = 2,168 m au millimètre près .

b. Vitesse : v = 36 km/h soit 10 m/s . La durée corres-pondant à une longueur de roue sur le sol est de :

2,168 ÷ 10 = 0,2168 s .

c. La période de la roue en rotation est donc T = 0,2168 s soit une fréquence de :

n = 1

0,216 8= 4,61 tr/s ou 277 tr/min .

d. Roue : n2 = 277 tr/min

D2

Pédalier : n1 = 80 tr/min à 100 tr/min

D1 = 50 dents

Pour n1 = 80 tr/min : D2 = 80 × 50

277= 14,4 .

Pour n1 = 100 tr/min : D2 = 100 × 50

277= 18,05 .

Pour rouler à 36 km/h avec ces fréquences de péda-lage, il peut utiliser les pignons de 15, 16 et 18 dents .

18. Scootera. Arbre moteur : n1 = 6 600 tr/min D1 = 90 mm

Variateur : n2

D1 = 70 mm

n2 = 6 600 × 90

70= 8486 tr/min .

b. Le rapport de transmission :

RZ

Zx

Z

ZT = = × =1

2

3

4

1352

1244

0 0682, .

Lorsque le variateur effectue 1 tour, la roue arrière effectue 0,068 2 tour .Lorsque l’arbre moteur fait 1 tour, le variateur effec-

tue

9070

tour et la roue arrière fait :

9070

0 0682 0 0877× =, , tour .

À 6 600 tr/min, la roue tourne à la fréquence de6 600 × 0,087 7 = 579 tr/min, soit à la vitesse de :

579 × π .D = 691 m/min ou 41,5 km/h .La vitesse maximale de 45 km/h est bien respectée .

19. Lecteur de CDa. Pour v = 1,2 m/s, la durée pour un déplacement sur une alvéole de 2,224 × 10-6 m est :

Δt = 2,224 ×10−6 ÷ 1,2 = 1,85×10−6 s .

b. Sur la première piste : R = 22 × 10-3 m et

nv

R= =

× ×=

−21 2

2 22 108 68

3π π .,

, tr/s .

Sur la dernière piste : R = 58 × 10-3 m et

nv

R= =

× ×=

−21 2

2 58 103 29

3π π .,

, tr/s .

Selon la position de la tête de lecture sur le disque, la fréquence de rotation du moteur varie .

20. Démarche d’investigationUn spectateur de la grande roue doit :- mesurer le diamètre approximatif de la grande roue en imaginant sa projection sur le sol (avec un mètre pliant) ;- chronométrer la durée mise par une nacelle pour faire un tour, à l’aide du chronomètre d’une montre électronique ;- utiliser la relation v = π .D .n, la fréquence de rota-tion n étant déduite de la période T précédemment mesurée .

Page 66: COLLECTION DURANDEAU SCIENCES